You are on page 1of 137

Test Information

Test Name INICET CBT-1 (Oct) 2022 Total Questions 216

Test Type Examination Difficulty Level Difficult

Total 180
200 Duration
Marks minutes

Test Question Language:-ENGLISH

1. A 59-year-old woman with pain at the side of her skull comes to the emergency department. An
emergent head CT scan shows a large lesion in the internal auditory meatus. This condition may
progress and damage which of the following pairs of structures?

a. Vagus and glossopharyngeal nerves

b. Internal carotid and vertebral arteries

c. Internal jugular vein and trigeminal nerve

d. Facial and vestibulocochlear nerves

Solution. (D Facial and vestibulocochlear nerves


Exp: Facial and vestibulocochlear nerves pass through internal auditory meatus.

Answer. d

2. A patient's left hypoglossal nerve (CN XII is injured during a surgical procedure. Which of the fol-
lowing would most likely result from this injury?

a. Decreased gag reflex on the left

b. Inability to elevate the pharynx on the left during swallowing

c. Deviation of the tongue to the left on protrusion

d. Deviation of the tongue to the right on protrusion

Solution. C. Deviation of the tongue to the left on protrusion


Exp: Healthy muscle with protrude the tongue and will push towards the injured side.

Answer. c
3. Parasympathetic fibers from CN IX synapse in which ganglion before traveling to the parotid gland?

a. otic ganglion

b. ciliary ganglion

c. submandibular ganglion

d. pterygopalatine ganglion

Solution. (A otic ganglion


Ref– Read the text below
Sol:
Ciliary – CN III
Submandibular ganglion – CN VII
Pterygopalatine ganglion– CN VII

Answer. a

4. Marked structure is supplied by which of the Following?

a. Spinal Accessory N.

b. C1 via Hypoglossal

c. Ansa Cervicalis

d. Vago Accessory complex

Solution. Marked structure is Thyrohyoid and is supplied by C1 via Hypoglossal N.

Answer. b
5. Correct sequence of coverings of Testes from outside inwards is:
1. External Spermatic Fascia
2. Internal Spermatic Fascia
3. Dartos
4. Cremastric Fascia

a. 3,2,1,4

b. 2,3,1,4

c. 3,1,4,2

d. 2,1,4,3

Answer. c

6. A patient presents with right leg numbness. When he walks, you notice that he lifts his right foot
higher than he does his left, and that his right foot slaps to the ground with each step. On neurologic
exam, it is found that he is unable to dorsiflex and evert the right foot as shown in pic. Which of the
following nerves has most likely been injured?

a. Tibial

b. Common peroneal

c. Superficial peroneal

d. Obturator

Solution. B Common peroneal


Exp: Common peroneal nerve injury is common and typically results from trauma to the leg near
the head of the fibula. Signs include foot drop and a characteristic high-stepping gait.

Answer. b
7. A 53 year old man is brought to emergency department for evaluation of fever, chills & malaise.
Cardiac examination reveals a new holo-systolic heart murmur that radiates towards the axilla. Blood
cultures are obtained & he undergoes transesophageal echocardiography. The ultrasound probe is
placed in the mid-esophagus facing anteriorly & cardiac chambers are interrogated.
Anatomically which chamber of heart is closest to the probe –

a. Right Ventricle

b. Left Ventricle

c. Right atrium

d. Left atrium

Solution. d. Left atrium


Exp: Transesophageal echocardiography evaluates all the chambers but Left atrium lies in front of
Oesophagus.

Answer. d

8. A 60-year-old man presents with history of headache, vertigo, ataxia and intermittent pain and
weakness in his left arm initiate by using the left arm for daily activities. On examination, the left radial
pulse is weak and the systolic BP on the left side is reduced by 30 mm HG. Doppler Ultrasound reveals
reversal of flow in the left vertebral artery. Critical stenosis of which artery would explain the patient’s
condition:

a. First part of Maxillary artery

b. Third part of Maxillary artery

c. First part of Subclavian artery

d. Third part of Subclavian artery

Solution. c. First part of Subclavian artery


Exp: This is the case of Subclavian steal syndrome which involves blocking of First part of
Subclavian artery.

Answer. c
9. which of the following marked nerves are responsible for eye movement

a. A&C

b. B&D

c. ABC

d. ACD

Solution. b. B&D
Exp: 3,4 &6 cranial nerve supply Extra ocular muscles.
A- 2nd cranial nerve
B- 3rd cranial nerve
C- 5th cranial nerve
D-4th cranial nerve

Answer. b
10. A known case of leprosy presented to the Dermatology OPD with the complaints of deformity in his
left hand as shown in the figure. Such a deformity is because of palsy of:

a. Ulnar nerve

b. Radial nerve

c. Both ulnar & median nerve

d. Median nerve

Solution. A.Ulnar nerve


Exp: Given pic shows partial claw also there is given history of leprosy so it’s a case of Ulnar N.
palsy.

Answer. a

11. A 28-year-old man reports to the physician because he and his wife have been unable to conceive
for the last 2 years. Evaluation for primary infertility is started and laboratory studies show a low
sperm count. A more thorough evaluation elucidates a rare genetic abnormality that leads to
permanently inactive FSH receptors. Which of the following substances is most likely decreased in this
patient?

a. FSH

b. LH

c. Inhibin B

d. Testosterone

Solution. C; Inhibin B
FSH receptors are on Sertoli cells. FSH acts on Sertoli cells stimulating the release of Inhibin B.
Inhibin B has negative feedback on the production of FSH.

Answer. c
12. A 20-year-old volunteer in a Physiology experiment was immersed in water up to the neck for a
period of 3 hours. Plasma concentration of different hormones were measured and recorded. One such
graph is given below.

Which of the following hormones is most likely to show changes in plasma concentration during and
after immersion in water?

a. PRA (plasma renin activity

b. ANP

c. Aldosterone

d. ADH

Solution. B; ANP
Immersion in water up to the neck removes the effect of gravity. This decreases the peripheral
pooling of blood and increases the effective circulating blood volume. Increase in blood volume
stimulates the atrial stretch receptors which increases ANP and decreases ADH causing a
natriuresis and a diuresis. Increase in renal afferent arteriolar pressure decreases renin and
therefore, aldosterone secretion.

Answer. b
13. Given below is graph showing the changes in tubular fluid/ plasma ratio of four different
substances along the length of the proximal tubule.

Which of the lines given in the above graph represents Inulin?

a. A

b. B

c. C

d. D

Solution. A
Inulin is a substance that is freely filtered, not reabsorbed, not secreted. Since two thirds of water
is reabsorbed in the PCT, the tubular fluid concentration of Inulin will increase. Thi s increases
the TF/P ratio for Inulin.
B shows a TF/P = 1.0. If the reabsorption of a substance is equal to the reabsorption of water in
the PCT, its TF/P ratio is equal to 1.0. This is osmolality or sodium.
For C and D, the TF/P ratio decreases along the length of the PCT. If the reabsorption of the
substance is more than the reabsorption of water along the length of PCT, its TF/P ratio will
decrease. C is bicarbonate and D is glucose or amino acids.

Answer. a
14. In a 37-year-old male, blood flow to a part of the lung is compromised. Which point on the V/Q line
of the O2- CO2 diagram given below corresponds to the alveolar gas of the distal alveoli?

a. A

b. B

c. C

d. D

Solution. C
If blood flow to a part of the lung is reduced the V/Q ratio in the distal alveoli increases. An
increase in the V/Q ratio increases the Po2 and decreases the Pco2 as at point C. At point D, the
V/Q ratio is equal to infinity, which causes the composition of the alveolar air to become equal to
that of the dead space air.

Answer. c
15. A healthy 35-year-old man hears the doorbell and suddenly stands up from his bed. Which of the
following cardiovascular changes is most likely to occur in response to standing up from the supine
position?
a. Increased sympathetic nerve activity
b. Increased heart rate
c. Increased parasympathetic nerve activity
d. Increased venous compliance
Use the following key to answer:-

a. Options a and b are correct

b. Options c and d are correct

c. Options a, b and c are correct

d. All are correct

Solution. A; options a and b are correct


Standing from the supine position causes an acute fall in the mean arterial pressure which is
sensed by the baroreceptors. Baroreceptor reflex leads to an increase in the sympathetic outflow
and decrease in parasympathetic outflow. Increased sympathetic and decreased parasympathetic
outflow to the heart results in an increase in heart rate. Increased sympathetic discharge to the
peripheral blood vessels causes an arteriolar constriction, increasing the TPR and therefore, MAP.
Increased sympathetic discharge to the veins causes a venoconstriction, decreasing venous
compliance and increasing venous return.

Answer. a

16. A 15-year-old boy is standing on a six feet high wall and suddenly jumps off landing on the balls of
his feet. The increase in muscle tension causes a sudden, complete relaxation of the affected muscles.
Which of the following statements is/are correct during relaxation of the muscles when tension is
increased?
a. Increased rate of discharge of muscle spindle
b. Centre for this reflex is the spinal cord
c. It a mono-synaptic reflex
d. Efferent arm of this reflex is Aα
Use the following key to answer:-

a. Options a and c are correct

b. Options b and d are correct

c. Options a, b and c are correct

d. All are correct

Solution. B; Options b and d are correct


The Golgi tendon organ senses the tension in the tendons. When the tension becomes exceedingly
high, a bi-synaptic inhibitory reflex is activated that causes relaxation of the entire muscle, which
protects the muscle from tearing and from avulsion of tendon. The centre for this reflex is the
spinal cord and the efferent arm of this reflex is the Aα motor neurons.

Answer. b
17. A model cell with three different transporters (X, Y and Z with RMP of -70mV is shown in the given
figure. Consider the intracellular and extracellular concentration of all ions to be typical of a normal
cell. Which of the following best describes transporter Z?

a. Primary active transport

b. Secondary active transport

c. Facilitated diffusion

d. Simple diffusion

Solution. D; simple diffusion


Transporter Z is a chloride channel, which is a simple diffusion of chloride through a water-filled
membrane channel.
Transporter X is the sodium- potassium ATPase pump which transports sodium and potassium
against their concentration gradients utilizing energy in the process.
Y transports sodium along its electrochemical gradient and calcium against its gradient out of the
cell. The energy required to move calcium against its concentration gradient comes form the
movement of sodium along its gradient which has been established by the activity of the Na+- K+
ATPase. This is an example of secondary active counter transport.

Answer. d
18. Which of the following would be expected to increase the mean systemic filling pressure?
a. Increased blood volume
b. Increased sympathetic discharge
c. Increased venous compliance
d. Increased parasympathetic discharge
Use the following key to answer

a. Options a, b, c are correct

b. Options a and b are correct

c. Options c and d are correct

d. Options a, b, c and d are all correct

Solution. B; Options a and b are correct


Mean systemic filling pressure or Psf is the pressure measured everywhere in the systemic
circulation after blood flow has been stopped by clamping the great vessels. Without blood flow
pressures everywhere in the systemic circulation become equal and this equilibrated pressure
level is the mean systemic filling pressure. Normal Psf is 7 mm Hg. Psf is the major pressure
affecting venous return. The two main determinants of Psf are (i blood volume, and (ii
sympathetic stimulation.
Mean systemic filling pressure will increase with increase in blood volume. Increased sympathetic
discharge will cause a decrease in venous compliance and increase the mean systemic filling
pressure increasing the venous return. Therefore, a decreased and not an increased venous
compliance will increase the mean systemic filling pressure. Parasympathetic stimulation has no
effect on mean systemic filling pressure.

Answer. b
19. A 36-year-old woman with a history of depression attempts suicide by locking herself in car with
the ignition on and the exhaust connected with a pipe to the cabin of the car. She is found comatose in
her car by her husband and is brought to the Emergency of a nearby hospital. Which of the following
OHDC corresponds to changes in the blood in this patient (normal OHDC in resting conditions is
depicted in blue?

a.
b.

c.

d.

Solution. C
The patient has CO poisoning. In Co poisoning the saturation of Hb is reduced and the OHDC is
shifted to the right.

Answer. c
20. A large dose of insulin is administered to a patient. Which of the following hormonal changes are
likely in this patient?
a. Increase in growth hormone
b. Increase in glucagon
c. Increase in epinephrine
d. Increase in aldosterone
Use the following key to answer

a. Options a, b, c are correct

b. Options a and b are correct

c. Options c and d are correct

d. Options a, b, c and d are all correct

Solution. A; options a, b and c are correct


Injection of Insulin leads to decrease in blood glucose. Hypoglycemia stimulates the secretion of
GH, glucagon and epinephrine, all of which have counter- regulatory effects to increase the
glucose levels in the blood. Hypoglycemia does not increase the aldosterone levels.

Answer. a

21. Following are the products of tyrosine


1 Melanin
2 Histamine
3 Norepinephrine
4 Melatonin

a. 1,2,3

b. 1,2

c. 1,3

d. 2,4

Solution. C : Tyrosinase a copper dependent enzyme convert tyrosine to melanin.


Catecholamines and thyroid hormones are other products.

Answer. c
22. Which of the following enzyme in heme synthesis pathway is inhibited in lead toxicity ?
1 Coproporphyrinogen oxidase
2 Ferrochelatase
3 ALA synthase
4 ALA dehydratase

a. 1,2,3

b. 3,4

c. 2,3,4

d. 1,2,4

Solution. D: Lead is a potent inhibitor of δ-aminolevulinic acid dehydratase (ALAD,


coproporophyrinogen oxidase, and ferrochelatase, enzymes that catalyze the second, sixth, and
final steps, respectively, in the biosynthesis of heme. Lead displaces zinc from the enzyme’s active
site, and the inactivation of ALAD has been implicated in the pathogenesis of lead poisoning. The
resulting accumulation of its substrate, ALA, has been shown to have a neuropathogenic effect,
probably by acting as a γ-aminobutyric acid (GABA receptor agonist in the nervous system.

Answer. d

23. Which of the following vitamins and deficiency disorders are correctly matched
1 Thiamine: lactic acidosis
2 Biotin: multiple carboxylase deficiency
3 Pantothenate: Acrodermatitis enteropathica
4 Pyridoxine: Neonatal convulsion

a. 1,2,4

b. 2,3,4

c. 1,4

d. 2,3

Solution. a:
• Thiamine is a cofactor of PDH complex.Pyruvate dehydrogenase complex (PDC is deficiencies
are inborn errors of carbohydrate metabolism that can profoundly impact the nervous system. In
PDH deficiency pyruvate cannot be converted to acetyl-CoA, resulting in fatal neonatal lactic
acidosis.
• Multiple carboxylase deficiency affects carboxylases The underlying disturbance involves
metabolism of biotin, a vitamin essential for the action of carboxylases.
• Thus, patients with pyridoxine-dependent epilepsy have an inactivated pyridoxal 5′–phosphate,
which is required for GABA synthesis and other vitamin B6–dependent enzyme reactions. GABA is
a critical inhibitory neurotransmitter. The CSF concentration of GABA is decreased in patients
with pyridoxine-dependent epilepsy.

Answer. a
24. Which of the following sequences is most likely to be a restriction enzyme recognition site?

a. CGGCTT

b. GTCGAC

c. GTAATG

d. GAATTG

Solution. B : Most restriction enzymes recognize palindromic sequences, meaning that both
strands of DNA will have the same sequence when read 5′ to 3′. For example, the sequence
GTCGAC is palindromic. Some other sites are
ATTGCAAT, 5′ GGATCC 3′ etc.

Answer. b

25. Collagen has unusual amino acid composition. The repeated tripeptide in collagen consists of “Gly-
X-Y”, where the X and Y represent

a. Lysine and proline respectively

b. Alanine and hydroxyproline respectively

c. Proline and hydroxyproline respectively

d. Lysine and hydroxylysine respectively

Solution. C: Collagen is composed of three individual left-handed helical polypeptides that


are wound tightly around each other to form a long, rope-like triple helix
molecule called procollagen.
Each individual left-handed collagen peptidecontains roughly 33% glycine (G, 13% proline (P, and
9% hydroxyproline (Hyp. Glycine is found in repeating sequences of Gly-X-Y, where the X position
is occupied by proline, and the Y position is filled by either proline or
lysine.

Answer. c
26. Which of the following is incorrect of structural organization of proteins

a. Protein denaturation does not affect primary structure

b. Proline is rarely found in alpha helix

c. Replacement of glutamic acid by valine in HbS results in defective tertiary structure of


haemoglobin

d. Elastin has unique triple helix secondary structure due to desmosine cross links.

Solution. d: Triple helix is characteristic of collagen. Elastin is synthesised and secreted in the
form of a precursor, tropoelastin, which is cross-linked with desmosine to form stable elastin. The
formation of desmosine requires the enzyme lysyl oxidase; defects in this enzyme are seen in
Menkes' syndrome.Elastin is present in alveoli, arteries and skin.

Answer. d

27. Malonate act as competitive inhibitor of

a. Succinate dehydrogenase

b. Succinate thiokinase

c. Cytochrome oxidase

d. Cytochrome reductase

Solution. a: Malonate is a reversible inhibitor of succinate dehydrogenase. Succinate


dehydrogenase plays a central role in the tricarboxylic acid cycle and as part of complex II of the
electron transport chain.

Answer. a
28. Deficiency of carnitine will result in

a. Defective alpha oxidation of phytanic acid

b. Zellweger syndrome

c. Accumulation of medium chain fatty acids in mitohcondria

d. Non ketotic hypoglycemia

Solution. d : Carnitine is produced endogenously from lysine and methionine. Beta oxidation of
the long-chain fatty acid (LCFAs occurs exclusively in the mitochondrial matrix. The mitochondrial
membrane is impermeable to LCFAs and requires the obligatory carnitine shuttle. In carnitine
deficient states, LCFAs cannot be effectively transported to the mitochondria matrix for oxidation
and subsequent utilization in Kreb’s cycle and ketone body production.
During periods of fasting, improper utilization of fatty acids impairs gluconeogenesis and
characteristically leads to nonketotic or hypoketotic (no or minimal ketone body production
respectively hypoglycemia.

Answer. d

29. Which of the following is not matched correctly

a. Klenow fragment: DNA polymerase which lack 5'→3' polymerase activity

b. eIF4E: identify the mRNA cap

c. Telomerase: reverse transcriptase

d. CRISPR-CAS9: genome editing tool

Solution. A : DNA Polymerase I, Large (Klenow fragment was originally derived as a proteolytic
product of E.coli DNA polymerase that retains polymerase and 3’ —> 5’ exonuclease activity, but
lacks 5’ —> 3’ exonuclease activity.

Answer. a
30. Glycogen storage disease presenting with mild hypoglycaemia, hepatomegaly and accumulation of
abnormally structured glycogen with small branches ?

a. Andersen disease

b. Cori disease

c. Pompe disease

d. Her’s disease

Solution. B
GSDIII and IV present with abnormal glycogen structure
GSD type III is also known as Forbes-Cori disease or limit dextrinosis, due to deficient
debranching enzyme .accumulated glycogen has abnormal partially degraded short chains or
small branches.
GSD type IV, also known as amylopectinosis, Glycogen Branching enzyme deficiency (GBE or
Andersen disease, present with an abnormal less branched and insoluble glycogen, which
precipitate.

Answer. b

31. True about mutations and their diagnosis is/ are?


I. Point mutations in exons can lead to Nonsense mutation
II. Point mutations in introns do not cause disease
III. Single base primer excision test is used for point mutations
IV. Frame shift are most severe mutations
V. Genetic anticipation is seen in trinucleotide repeat mutations

a. III,IV,V

b. II,III,IV,V

c. I,III,IV,V

d. All are true

Solution. C. point mutations in introns cause splicing defects thus causing diseases Eg
Thalassemia

Answer. c
32. Electron microscopy of a patient with lytic lesions in bone is as follows. Not true about the disease
is?

a. EM is diagnostic of this disease

b. It is a hematological neoplasm

c. Can present with diabetes insipidus

d. Predominantly occurs in elderly

Solution. D. the EM shows Birbeck granules seen in Langerhan cell histiocytosis, which is a
disease of children not elderly.

Answer. d
33. True about to Warburg effect is/are?
I. Increased ATP production by tumor cells
II. Increased glucose consumption by tumor cells
III. Happens in viral infected cells
IV. Shift of glucose metabolism to Anaerobic glycolysis
V. Happens in cancer cells

a. II,III,V

b. II,IV,V

c. I,II,IV,V

d. I,II,III,V

Solution. A. Warburg effect is shift to Aerobic glycolysis, it happens in any replicating cell
including viral infected cells, stem cells repair regeneration, embryogenesis and cancer cells. Due
to Warburg effect there is reduced ATP production to compensate this Warburg effect leads to
increased glucose consumption by these cells.

Answer. a

34. Which of the following is correct chronological order of appearance iron deficiency anemia and its
treatment?
I. Increased serum ferritin
II. Increased hemoglobin
III. Increased reticulocyte
IV. Decreased serum ferritin
V. Decreased serum Iron

a. IV,V,I,III,II

b. IV, V, III, I,II

c. IV,V, III, II,I

d. V,IV,III,II,I

Solution. C. options IV and V are occurring in disease (iv followed by V and options I,II,II occur in
response to oral iron in treatment ( order III,II,I

Answer. c
35. A 45-year painter presents with easy fatigue and abdominal pain for last 8 months, he has been
taking iron and vitamin supplements for last 6 months but without any improvement. blood
examination shows Hb 8g/dl, MCV 68, MCH 22, Retic count 1.1% serum iron is 500μg/dL and serum
ferritin 512μg/L. which of the following is most likely diagnosis?

a. Thalassemia minor

b. Lead poisoning

c. B12 deficiency anemia

d. Anemia of chronic disease

Solution. B. lead poisoning. Microcyti anemia not responding to iron and serum ferritin and
serum iron are both increased which is seen in sideroblastic anemia, one of the causes is lead
poisoning

Answer. b

36. Histological examination of a patient suffering from diarrhoea and malabsorption syndrome is as
follows. What is the likely etiology?

a. Infectious

b. HLA Dq2 association

c. Inherited MTP mutation

d. Autoimmune

Solution. A. the image shows foamy macrophages in the lamina propria which is characteristic of
Whipple disease caused by T whippeli.

Answer. a
37. Which of the following features is seen in Nephritic syndrome but not in nephrotic syndrome?

a. Lipiuria

b. Renin Angiotensin activation

c. Progression to acute renal failure

d. All the above

Solution. C. progression to ARF is seen in nephritic syndromes in which there is reduced urine
output which can progress to ARF (known as Rapidly progressive glomerulonephritis

Answer. c

38. An elderly retired person presents with recent onset of breathlessness. He has developed new
hobby of feeding grains to pigeons. His FEV1/FVC ratio is normal. Histological examination from lung
shows non necrotizing granulomas Which of the following is most likely pathogenesis?

a. Type I hypersensitivity

b. Autoimmune

c. Type III hypersensitivity

d. Type IV hypersensitivity

Solution. D. the patient is possibly having a pigeon breeders’ lung which is hypersensitivity
pneumonitis, which is predominantly type IV hypersensitivity reaction.

Answer. d
39. A 65-year female presents swelling in the inguinal region for last two weeks. Examination shows
multiple palpable inguinal lymphnodes. A biopsy was taken. Histology showed large pleomorphic cells
with prominent nucleoli and intracellular black pigment in the cells as seen in the image. What is the
likely diagnosis?

a. Metastatic squamous cell carcinoma

b. Hodgkin Lymphoma

c. Metastatic melanoma

d. Endometriosis

Solution. C. Metastatic melanoma. The key word here is black pigment inside the neoplastic cells
which is melanin, seen in melanoma.

Answer. c

40. Which of the following disease of breast is most premalignant?

a. Radial scar

b. Adenosis

c. Intraductal papilloma

d. Atypical duct hyperplasia

Solution. D. atypical duct hyperplasia. Radial scar comes under proliferative disease without
atypia less premalignant to proliferative disease with atypia.

Answer. d
41. Histological examination of thyroid tumor is as follows. What is the likely diagnosis?

a. Papillary carcinoma

b. Follicular carcinoma

c. Medullary carcinoma

d. Lymphoma

Solution. C. there is typical amyloid seen in the image (pink acellular material typically seen in
medullary carcinoma

Answer. c

42. A 54-year male had an aortic regurgitation and died due to complications of the same. A medical
autopsy was done to identify the changes in the heart. Which of the following is the best measure to
arrive at a conclusion of hypertrophy of this patient’s heart?

a. Increased diameter of left ventricle

b. Increased thickness of left ventricular free wall

c. Increased weight of the heart

d. Increased thickness of interventricular septum

Solution. C. Increased weight of the heart. There can be increased in thickness of the ventricular
free wall in hypertrophy (in concentric type, in eccentric type of hypertrophy the sarcomeres are
arranged in series thus thickness does not increase thus weight of the heart becomes the best
parameter to identify hypertrophy in such cases.

Answer. c
43. The correct order of adenoma carcinoma sequence in a colorectal carcinoma is
i. K-RAS
ii. APC mutation
iii. P53
iv. Histone methylation

a. ii,i,iv,iii

b. ii,iv,i,iii

c. i,ii,iv,iii

d. i,iv,ii,iii

Solution. B. ii, iv,I,iii. The correct order of sequence is first hit of APC- DNA methylation of
normal allele (2nd Hit-KRAS mutation-P53, telomerase and other mutations

Answer. b
44. As an intern you are asked to send samples for blood grouping and cross matching of a patient,
which of the following is most appropriate order of vacutainer for taking sample?

a. Red followed by purple

b. Purple followed by red

c. Only red

d. Only purple

Solution. A.

Answer. a
45. Match the following special stains with their utility

Stain Used in

a. Masson fontana p. Melanin

b. Masson trichrome q. Hemosiderin

c. Prussian blue r. Collagen

d. Alcian blue s. Mucin

t. Muscle

Answer. a-p,b-r,c-q,d-s

46. Choose the correct Toxin & Mechanism of Action

a. Pertussis toxin p. Acting of EF2 & Inhibits protein synthesis

b. Diphtheria toxin q. Acting on 28sRibosome & inhibits protein

c. Shiga toxin r. Inhibits the release of GABA and Glycine

d. Botulinum toxin s. Increasing cAMP

t. Inhibits the release of Acetyl choline

u. Somatic antigen

Solution. D
i. Pertussis toxin - Increasing cAMP
ii. Diphtheria toxin - Acting of EF2 &
Inhibits protein synthesis
iii. Shiga toxin - Acting on 28sRibosome
& inhibits protein synthesis
iv. Botulinum toxin - Inhibits the
release of Acetyl choline

Answer. a-s,b-p,c-q,d-t
47. List of helminths are given below. Egg is the infective stage in which of the following helminths.
1. Hymenolepsis nana,
2. Taenia saginata,
3. Echinococcus granulosus,
4. Diphyllobothrium latum,
5. Strongyloides stercoralis,
6. Enterobius vermicularis

a. 1,2,3,4 are correct

b. 1,3,4 are correct

c. 1,3,6 are correct

d. 1,4,5 are correct

Solution. C
• Hymenolepsis nana, Echinococcus granulosus and Enterobius vermicularis - Egg is infective
stage
• Taenia saginata Diphyllobothrium latum and Strongyloides stercoralis – Larva is infective stage.

Answer. c

48. Choose the correct statement(s regarding Rubeola.


1. Koplik spot appears in prodromal stage,
2. Vaccine strain is Jeryl Lynn strain,
3. Rash first appears on the trunk,
4. Vaccine – at 9 month,
5. Fever and rash is the current case definition,
6. Incubation period 2-3 days

a. 1,2,3,4 are correct

b. 1,3,4 are correct

c. 1,3,6 are correct

d. 1,4,5 are correct

Solution. D
• Rubeola is the other name for measles
• Koplik spot appears in prodromal stage
• Vaccine strain is Edmonston Zagreb strain
• Rash first appears behind the ears
• Vaccine – at 9th month
• Fever and rash is the current case definition
• Incubation period – 10 days

Answer. d
49. Choose the correct statement(s regarding the clinical condition of a STI shown in the image.

1. Ulcer bleed easily,


2. Painful,
3.Bubo formation,
4. Typically indurated,
5.Caused by H. ducreyi,
6. Hard chancre

a. 1,2,3,5 are correct

b. 1,3,4 are correct

c. 1,3,6 are correct

d. 1,4,5 are correct

Solution. A
• It’s a case of Chancroid.
• Ulcer bleed easily.
• Painful.
• Bubo formation.
• Typically, non indurated
• Caused by H. ducreyi.
• Soft chancre
• Treatment – Azithromycin

Answer. a
50. A patient of Acute lymphocytic leukemia with fever and neutropenia develops diarrhea after
administration of amoxycillin therapy. Choose the correct selective medium for this pathogen?

a. EMJH agar

b. CCF Agar

c. PLET agar

d. BCYE agar

Solution. B
• It’s a case of antibiotic associated pseudomembranous enterocolitis caused by Clostridioides
difficile .
• Selective medium is Cycloserine Cefoxitin Fructose Agar.
• DOC is Fidaxomycin
• EMJH agar - Leptospira
• PLET agar – Bacillus anthracis
• BCYE agar – Legionella pneumophila

Answer. b
51. A farmer had a flea bite while working in a farm. Five days later he developed fever and had
axillary lymphadenopathy. A smear was sent to the laboratory to perform a specific staining. The
pathogen is safety pin appearance as shown in the image. Choose the correct staining method for this
pathogen.

a. Albert staining

b. Ziehl – Neelsen staining

c. McFadyean’s staining

d. Wayson staining

Solution. D
• Farmer: High-risk of exposure to rodents and history of flea bite. (Rat flea
• Presented with high fever, painful axillary lymphadenopathy suggestive of plague caused by Y.
pestis.
• Special stain is Wayson stain - When stained with a polychromatic stain (e.g. Wayson, Y. pestis
isolated from clinical specimens exhibits a characteristic bipolar appearance, resembling closed
safety pins.
• Albert’s stain: Used to demonstrate metachromatic granules in C. diphtheriae
• McFadyean’s stain: Used to demonstrate polypeptide capsule of B. anthracis
• Ziehl- Neelsen stain for Acid fast organisms.

Answer. d
52. A 30-year-old patient presented with history of jaundice for 10 days. His liver function tests showed
bilirubin of 10 mg/dl, SGOT/SGPT – 1100/1450, serum alkaline phosphatase – 240 IU. He was positive
for Hbs Ag. Choose the correct confirmatory test to establish acute hepatitis B infection?

a. IgM anti-HBc antibody

b. HbeAg

c. HBV DNA by PCR

d. IgG Anti-HBc antibody

Solution. A
• Anti-HBc IgM is the first antibody to elevate following infection. • It appears within the first 1–2
weeks after the appearance of HBsAg and lasts for 3–6 months.
• Its presence indicates acute hepatitis B infection. It is probably the only marker (sometimes
anti-HBc IgG present during the period between appearance of anti-HBs antibody and
disappearance of HBsAg (Window Period.
• Anti-HBc IgG appears in late acute stage and remains positive indefinitely whether the patient
proceeds to Chronic stage (with persistence of HBsAg, symptomatic and elevated liver enzymes
and Carrier state (with persistence of HBsAg but asymptomatic or Recovery (appearance of Anti-
HBs antibody.
• HBV DNA & HBeAg - Active viral replication and High viral infectivity.

Answer. a

53. Which one of the following is detected by the antigen detection test used for the diagnosis of P.
falciparum malaria?

a. Circum –sporozoite protein

b. Merozoite surface antigen

c. Histidine –Rich-Protein I

d. Histidine – Rich –Protein II

Solution. D
• Antigen detection tests / Rapid diagnostic tests / Immunochromatographic tests.
• Rapid and simple.
• pLDH and Aldolase: Plasmodium genus specific.
• HRP-2 Ag detection: Specific for P.falciparum

Answer. d
54. A 30-year-old man came to OPD with Itch in his legs. This appears after swimming and the clinician
diagnosed it as swimmers itch caused by Schistosoma hematobium. All the following are the mediators
of this reaction except

a. 5-hydroxytryptamine

b. Prostaglandin

c. C3a & C5a

d. Platelet activating factor

Solution. B
• Swimmers itch caused by S. hematobium belongs to type I HSR – Anaphylaxis.
• Mediators for Type I HSR
• Primary mediators are Histamine, Heparin, Eosinophil and Neutrophil chemotactic factor
Proteases.
• Secondary mediators : Leukotrienes, slow reactive substance of anaphylaxis (SRS-A,
Prostaglandins, Cytokines (IL-1 and TNF-α, Anaphylatoxins and PAF.

Answer. b
55. A 47-year-old man initially comes to his primary care physician with persistent fever, night sweats,
and fatigue. Thorough evaluation yields a diagnosis of chronic myeloid leukemia. While undergoing
treatment for his malignancy, the patient comes to the oncologist complaining of headaches, scanty nasal
discharge, and a problem with his left eye. Physical examination reveals tenderness over the paranasal
sinuses in addition to left-sided orbital swelling and cellulitis. Mild proptosis and ptosis of the left eye are
also present. Biopsy of his sinus mucosa is shown below. Choose the correct pathogen.

a. Histoplasma capsulatum

b. Aspergillus fumigatus

c. Coccidioides immitis

d. Rhizopus species

Solution. B
• This patient's symptoms are consistent with fungal rhinosinusitis.
• The biopsy image shows fungal hyphae branching at acute angles with septations, which is
characteristic of Aspergillus fumigatus. This fungus is widely distributed in the environment and
commonly grows on decaying vegetables.
• It is monomorphic, existing only in mold form (ie, multicellular hyphae.
• Galactomannan assay for invasive aspergillosis.
• Treatment :Mild – Voriconazole, Severe - AmphoB

Answer. b
56. Table A contains the predominant snake venom of snakes belonging to elapidea, viperidea,
hydrophidea and non-poisonous snakes. Match it correctly with morphological features identifying
them, given in table B?

Table A Table B

a. Neurotoxic p. Multiple rows of scales on belly

b. Vasculotoxic q. Oar like tail

c. Myotoxic r. 4th infra-labial scale in largest

d. Non toxic s. Small head scales

Solution. C
• There chief poisonous species of
snakes in India and their three families
:
– Elapidea – includes common cobra,
king cobra and krait.
– Viperidae – russell’s viper, pit viper &
saw scaled viper.
– Hydrophidae – Sea snake

Answer. a-r,b-s,c-q,d-p

57. Dr Sandeep consumes alcohol and operates. The procedure goes well and pt is shifted to post-
operative care. One OT staff reports the incident to the police. What is the legality?

a. Doctor can not be arrested as pt is fine

b. Doctor can be arrested under sec 336 IPC

c. Doctor can be arrested under sec 304 IPC

d. Doctor will be arrested under sec 323 IPC

Solution. B
Sec 336 IPC- punishment for endangering someone’s life because of negligent
Action.
Punishment- 3months of imprisonment or Rs 250 fine or both.

Answer. b
58. The best tooth for age estimation by Gustafson’s method is?

a. 3rd molar

b. 1st molar

c. Central incisor

d. Canine

Solution. C
Central teeth are more accurate in Gustafson’s calculation and lateral teeth are not.
Bets is central incisor, worst is 3rd molar.

Answer. c

59. Three groups of subjects were followed over the course of five years to compare treatments for
sideroblastic anemia. The most appropriate statistical analysis to determine the quantitative serologic
differences in mean levels resulting from these treatments would be :

a. Student’s ‘t’ test

b. Paired ‘t’ test

c. ANOVA

d. Chi‐square test

Solution. C
ANOVA test : to compare quantitative data : > 2 groups

Answer. c
60. Which of the following is true regarding the study design shown ( below in the image :

a. It's not a cross over RCT

b. One set of patients will receive intervention only

c. Drop outs cannot be seen

d. Every patient can act as his own control while analysing the results

Solution. D
o It's a cross over RCT
o Both set of patients will receive intervention
o Drop outs can be seen
o Every patient can act as his own control while analysing the results

Answer. d
61. Identify correctly matched set for coloured bins used for biomedical waste disposal :

a. Metallic implants p. Red

b. Metallic sharps q. yellow

c. Amputed limbs r. White

d. Urine bags s. Blue

Solution. D
• Metallic implants : Blue box
• Metallic sharps: White
• Amputed limbs : Yellow
• Urine bags: Red

Answer. a-s,b-r,c-q,d-p

62. Prophylactic disinfection includes -


1. Disinfection of urine specimen of patient with enteric fever
2. Pasteurization of milk
3. Disinfection of contaminated linen
4. Disinfection of water by chlorine
Which of these statements are correct?

a. 1, 2, 3 and 4

b. 2 and 4 only

c. 1, 2 and 4 only

d. 1 and 3 only

Solution. B
• Disinfection of urine specimen of patient with enteric fever: Concurrent disinfection
• Pasteurization of milk : Prophylactic disinfection
• Disinfection of contaminated linen : Concurrent disinfection
• Disinfection of water by chlorine : Prophylactic disinfection

Answer. b
63. A trial was done on hypertensives to assess the protective effect of using an aspirin compared to
placebo. The information below ( in the table shows the number of patients experienced MI in aspirin
group compared to placebo. Calculate relative risk :

a. 1.5

b. 0.5

c. 0.8

d. 2.5

Solution. B
Risk in aspirin group : 100/1000
Risk in placebo group : 200/1000
Risk ratio : ½ = 0.5

Answer. b

64. Which of the following is/are false statements :


1. Pentavalent vaccine: 1st dose - can be given till 3yr
2. Rotavac is killed vaccine used in NIS
3. Vitamin A prophylaxis is given till 3yr
4. Pregnancy is not a contraindication for PEP in rabies

a. 1 only

b. 1,4

c. 1,3 only

d. 1,2,3

Solution. D
• Pentavalent vaccine: 1st dose - can be given till 1 yr
•Rotavac is live vaccine used in NIS
•Vitamin A prophylaxis is given till 5 yr
• Pregnancy is not a contraindication for PEP in rabies

Answer. d
65. Match the following for purpose of schemes for child health under RMNCH+A:

a. HBNC p. To train MOs for neonatal care

b. RBSK q. Home visits by ASHA

c. NSSK r. Care units at health facilities

d. FBNC s. Screen for 4Ds

Solution. B
• HBNC : Home visits by ASHA
• RBSK : Screen for 4Ds
• NSSK : To train MOs for neonatal
care
• FBNC: Care units at health facilities

Answer. a-q,b-s,c-p,d-r

66. Which of the following is true regarding the study findings as shown below in the table :

a. Findings of Study A is significant

b. Findings of Study B is significant

c. Study C is showing significant negative association

d. Study C is showing significant positive association

Solution. D
• Confidence limits of studies A and B has 1 in the range . Not significant
• Study C is showing significant positive association

Answer. d
67. An occupational worker presented with complaints of exertional dyspnea. He gave history of being
in an industry dealing with spare parts such as gas kit and brakes. His X-ray chest showed a ground
glass appearance / honey combing in the lower two thirds of the lung fields. The likely condition that he
suffers from is:

a. Silicosis

b. Anthracosis

c. Asbestosis

d. Siderosis

Solution. C
Patient is suffering with characteristic features of asbestosis

Answer. c

68. A pregnant from remote area of karnataka was diagnosed with malaria due to plasmodium
malariae in second trimester . Drugs for management as per malaria drug policy -

a. Quinine

b. Artesunate + SP

c. Artemether + Lumefantrine

d. Chloroquine

Solution. B
ACT is used. Artesunate + SP is used in second trimester for states other than north east

Answer. b

69. A 39-year-old man who presents with a mild sore throat, fever, malaise, and headache is treated
with penicillin for presumed streptococcal infection. He returns after week with hypotension, fever,
rash, and abdominal pain. He responds favorably to chloramphenicol, after a diagnosis of Rocky
Mountain spotted fever is made. This study is:

a. Case series

b. Case-control study

c. Clinical trial

d. Case report

Solution. D
Case report : To study one patient in detail

Answer. d
70. A woman brought her child with congenital anomalies of heart and cataract. She gave history of
mild fever and rash in the first trimester of pregnancy, which settled with mild antipyretics. The strain
of vaccine used for prevention of such anomalies:

a. SA 14-14-2

b. RA 27/3

c. 17D

d. Danish 1331 strain

Solution. B
RA 27/3 : Live vaccine is used for preventing congenital rubella syndrome

Answer. b

71. Nutritional research team followed serum levels of vitamin B12 in 120 children for three years to
determine the association between cyanocobalamin deficiency and the subsequent risk of developing
Megaloblastic anemia. The results were as follows:
VITAMIN B12 LEVELS: Mean - 260 pg/mL Median- 226 pg/mL Mode- 194 pg/mL
From the data, it can be concluded that this distribution is:

a. Normal distribution

b. Negatively skewed data

c. Positively skewed data

d. Bimodal data

Solution. C
Mean>Median>Mode : Positively skewed data

Answer. c
72. Which of the following is/are false statements regarding life cycle of infection shown below:

1. Clinical features: Acute viral hemorrhagic fever


2. SA 14-14-2 is used for prevention in all states
3. Vector: Culex rests indoor
4. Pigs are asymptomatic during amplification

a. 1 only

b. 1,2 only

c. 1,2,3,4

d. 1,2,3 only

Solution. D
• Clinical features: Acute encephailitis
• SA 14-14-2 is used for prevention in endemic states
•Vector: Culex rests outdoor
• Pigs are asymptomatic during amplification

Answer. d
73. Which of the following is true regarding anterior epistaxis?

a. Patient is kept in sitting and slight leaning forward position while compressing the nares

b. Topical phenylephrine is contraindicated as it increases blood pressure

c. Commonly from Kiesselbach venous plexus in the anterior septum

d. Brown’s Area is arterial in origin

Solution. (a Patient is kept in sitting and slight leaning forward position while compressing the
nares
Ref: Read the text below
Sol:
•Phenylephrine being a decongestant is used to prepare the before cauterization.
•Brown’s Area is venous in origin

Answer. a
74. Which of the following statement are true regarding the disease depicted here?
a. There are many macrophages with clear to foamy cytoplasm that harbor Klebsiella rhinoscleromatis
b. It is case of granulomatosis with polyangitis
c. It is caused by a gram-positive bacillus
d. Nasal obstruction is a common complaint
e. Streptomycin and tetracycline for 4-6 weeks are used in treatment

a. I, IV, V
b. I, II, III

c. I, III, V

d. III, IV, V

Solution. (a I, IV, V
Ref: Read the text below
Sol:
• This is a patient of rhinoscleroma caused by klebsiella rhinoscleromatis which is a gram negative
bacteria.
• Granulomatosis with polyangitis is wegner’s granulomatosis

Rhinoscleroma
Characteristic (submucosal granulomatous infiltrate composed of macrophages with clear to foamy
cytoplasm (Mikulicz cells intimately associated with an admixture of lymphocytes and plasma cells.

Answer. a
75. What is not true about Toss and Sade’s classification of retraction of tympanic membrane—

a. Tos classification grades retraction of pars flaccida

b. Sade classification grades retraction of pars tensa

c. Grade 4 Sade’s classification is atelectatic otitis media

d. Grade 4 Tos classification indicates severe erosion of outer attic wall

Solution. (c Grade 4 Sade’s classification is atelectatic otitis media


Ref: Read the text below
Sol:
Toss classification is for retraction of pars flaccid and Sade classification is for retraction of pars
tensa
Sade classification of retraction of pars tensa
Grade 1 – mild retraction
Grade 2—severe retraction—retracted TM touching incus or stapes
Grade 3—Atelectatic TM—TM touching promontory but moves on seigelisation
Grade 4---Adhesive TM--TM touching promontory, does not move on seigelisationos classification
of pars flaccida
Tos classification of pars flaccida
Stage I: Pars flaccida is dimpled and is more retracted than normal. It is not adherent to the
malleus.
Stage II: In this stage the retraction pocket is adherent to the handle of malleus. The full extent of
the retraction pocket can be clearly seen.
Stage III: In this stage part of the retraction pocket may be hidden. There may also be associated
erosion of the outer attic wall (scutum.
Stage IV: In this stage there is definite severe erosion of the outer attic wall. The extent of the
retraction pocket cannot be clearly seen as most of it are hidden from the view.

Answer. c
76. Study the given image and comment on the TNM staging of the patient :

a. T2NOMO

b. T2N1MO

c. T3N1MO

d. T4N1MO

Solution. (d T4N1MO
Ref: Read the text below
Sol.

Answer. d
77. A 6-month-old child developed labyrinthitis following meningitis. He was subjected to the test
shown below.What is this test being performed:

a. OAE

b. BERA

c. Tympanometry

d. PTA

Solution. b BERA
Ref: Read the text below
Sol: •BERA is useful both for screening and for quantitative audiometry in the high risk infant.

Answer. b
78. All of the following are true about the prosthesis shown below except

a. It carries air from trachea to esophagus

b. Used for laryngectomised patients for voice production

c. Aspiration of food into trachea is a frequent issue associated with its use

d. Speech quality is very good

Solution. (c Aspiration of food into trachea is a frequent issue associated with its use
Ref: Read the text below
Sol:
• Blom singer prosthesis is a tracheoesophageal puncture device.
• It is a one way valve surgically placed between trachea and esophagus.
• It is used for vocal rehabilitation after laryngectomy.
• It sends air from trachea to esophagus which is used to produce voice.
• But it does not let the food particles to go from esophagus to trachea so aspiration is not
significant

Answer. c
79. A 40 yrs old patient presented in emergency with vertigo, dizziness with horizontal nystagmus. Slow
component of nystagmus was towards left side. Comment on the most appropriate diagnosis. :

a. Superior canal BPPV

b. Posterior canal BPPV

c. Right hypoactive labyrinth

d. Left hypoactive labyrinth

Solution. (d Left hypoactive labyrinth


Ref: Read the text below
Sol.

Unilateral Vestibular hypofunction:


The eyes drift (slow phase toward the lesioned side and quick phase beat toward the intact side.

Answer. d
80. A previously healthy 32 years old female presented with rotatory vertigo associated with nausea
and vomiting. The symptoms had started five days earlier as short lasting vertigo attacks, her
symptoms had been continuous for three days. Clinical examination showed BP 200/95. Otoscopy was
normal No nystagmus in a neutral position, and no pathological findings by standard clinical
neurological examination was found. The next day the patient was still nauseated and described
continuous vertigo, with a slight improvement. Hallpikes manoeuvre did not evoke nystagmus or
exacerbation of her vertigo. Identify the investigation being done in the given image?

a. Cortical evoked response audiometry.

b. Brainstem evoked response audiometry.

c. Caloric Test.

d. Vestibular Evoked Myogenic potential.

Solution. (d Vestibular Evoked Myogenic potential.


Ref: Read the text below.
Sol:-
• In VEMP, we stimulate the saccule with sound and we record the contractions of
Sternocleidomastoid muscle ( Vestibulocollic reflex.
•This test checks the inferior vestibular division of 8th nerve as saccule is supplied by this nerve.

Answer. d
81. Incongrous homonymous visual field defects is seen in lesion of?

a. Optic nerve

b. Optic chiasma

c. Optic tract

d. Optic radiations

Solution. c
Retrochiasmal lesions are mostly contralateral homonymous
Optic tract can be congrous and incongrous

Answer. c
82. This is fundus picture of 25 year old female (gymnast by profession presented for routine
examination in Ophthalmology OPD. What is the probable diagnosis

a. Angioid streaks

b. Morning glory anomaly

c. Papilledema

d. Optic disc drusen

Solution. a
Angioid streaks result from crack-like breaks in Bruch's membrane, which is abnormal in its
structural composition, predisposing to these localized areas of rupture. This process may occur
spontaneously or could be secondary to blunt trauma, even very minor. Angioid streaks can be
associated with numerous systemic diseases, the most common being pseudoxanthoma elasticum.
A popular mnemonic used to recall the more common associations is PEPSI(mnemonic:

Answer. a
83. All are remnants of the hyaloid vasculature system except

a. Bergmeister's papilla

b. Persistent pupillary membrane

c. Mittendorf's dot

d. Posterior lenticonus

Solution. d
Hyaloid System (primary vitreous is composed of both the hyaloid vessel that extends from the
optic nerve to the posterior lens, as well as the vasculature that fills the vitreous cavity.
Remnants of the hyaloid vessels can form :
•Bergmeister papilla, seen as a tuft at the optic disc
• Mittendorf dot on the posterior lens surface
• Persistent hyperplastic primary vitreous
• Muscae volitantes
• Persistent pupillary membrane

Answer. d
84. In which part of eye this procedure is done?

a. Iris

b. Cornea

c. Anterior chamber

d. Lens

Solution. b
Corneal inlays, also called keratophakia, are implants placed in the corneal stroma for correction
of presbyopia, a condition in which the ability to accommodate or focus at near objects is
decreased.
Keratoconus is a contraindication for corneal inlays

Answer. b
85. Principle of Goldmann Applanation Tonometry is?

a. Michelson interferometer

b. Snells law

c. Imbert Fick law

d. Geometry of a spherical reflecting surface.

Solution. c
Applanation tonometry is based on the Imbert-Fick principle, which states that the pressure inside
an ideal, dry, thin-walled sphere equals the force necessary to flatten its surface divided by the
area of flattening (P = F/A, where P = pressure, F = force and A = area.
The functional principle behind Optical Coherence Tomography is Michelson interferometry.
Snell's law ( Law of refraction relates the incidence and refraction angles to the refraction indices
of the media involved.

Answer. c
86. The following sign can be seen in which condition

a. Congenital dacryocystitis

b. Congenital glaucoma

c. Congenital cataract

d. Congenital dermoid

Solution. b
Habb striae in congenital glaucoma

Answer. b
87. Causes of Cystoid Macular edema include all except

a. Post cataract extraction with intraocular lens

b. Prostaglandin use

c. Age Related Macular Degeneration

d. Retinal vein occlusion

Solution. c
Causes of Cystoid Macular edema:
• Post cataract extraction with intraocular lens (Irvine-Gass
•Pars planitis
• Vein occlusion
• Retinitis Pigmentosa
• Prostaglandin analogue
• Niacin

Answer. c
88. An infant was referred to ophthalmology OPD from ENT department, where they diagnosed him
with deafness. O/E Fundus photo is as shown below. What is the most likely diagnosis

a. Usher syndrome

b. Refsum syndrome

c. Abetalipoproteinemia Bassen-Kronzweig syndrome

d. Lawrence-Moon-Biedel syndrome

Solution. a
Systemic associations with Retinitis pigmentosa:
Usher’s syndrome: Congenital sensory neural deafness with RP
Kearns-Sayre syndrome (mitochondrial myopathy Pigmentary degeneration of the retina, external
ophthalmoplegia and complete heart block.
Refsum’s syndrome (phytanic acid storage disease RP with peripheral neuropathy, cranial
neuropathy, cerebellar involvement, cardiomyopathy and sudden death.
Abetalipoproteinemia Bassen-Kronzweig syndrome RP with infantile steatorrhea and failure to
thrive.

Answer. a
89. Which of the following neurodegenerative conditions caused the pathological findings seen in the
figure?

a. Alzheimer disease

b. Pick disease

c. Parkinson disease

d. Huntington disease

Solution. D. Huntington disease.


The image demonstrates atrophy of the caudate, with so-called boxcar ventricles, which is
characteristic of Huntington disease. Alzheimer disease may show atrophy of the mesial temporal
lobe and temporoparietal junction. Pick disease would show frontal atrophy. Parkinson disease
would show loss of pigmented neurons in the substantia nigra.

Answer. d
90. All of the following are features of critical illness polyneuropathy and myopathy except?

a. Intensive care unit (ICU stay longer than 2 weeks

b. Distal-to-proximal gradient of symptoms

c. Hyperglycaemia is a risk factor

d. Facial weakness

Solution. D. Facial weakness.


Critical illness polyneuropathy and myopathy are two separate disorders that frequently coexist.
These occur in the setting of a prolonged ICU stay and are present in 50% of patients who have
been in the ICU for longer than 2 weeks. These syndromes make it difficult for patients to wean
from the ventilator and can prolong their hospital stay. Hyperglycaemia and use of IV steroids are
also risk factors for the development of these conditions. There is usually a distal-to proximal
gradient of weakness, which can be shown on electromyography and nerve conduction studies. If
the patient survives the acute hospital stay, most recover in 3–6 months. Facial weakness is not
seen in either
condition and should point toward a different diagnosis, like Guillain-Barré syndrome,
neuromuscular junction disorders, or possibly a central lesion

Answer. d
91. An 8-year-old male presents to the office for evaluation of seizures. His parents state that he has
reached all developmental milestones and does well in school. In the last 2 months he has had several
nocturnal episodes with twitching of one side of his face and retained consciousness. A sample from his
electroencephalogram (EEG is shown. Which of the following is the most likely diagnosis?

a. Benign epilepsy with centrotemporal spikes

b. Autosomal-dominant nocturnal frontal lobe epilepsy

c. Juvenile absence epilepsy

d. Juvenile myoclonic epilepsy

Solution. A. Benign epilepsy with centrotemporal spikes.


The EEG and clinical scenario are most consistent with benign epilepsy with centrotemporal
spikes (BECTS, formerly known as benign Rolandic epilepsy. In this syndrome, seizures typically
start between 5 and 8 years of age and resolve in the early to middle teens. These episodes usually
involve paraesthesia of the face followed by clonic movements of the mouth. This can impair the
ability to speak, but not usually consciousness. Most seizures arise out of sleep and can
secondarily generalize on occasion. The EEG demonstrates bilateral independent centrotemporal
spikes with a horizontal dipole during sleep, best seen on a referential montage. Most epileptic
spikes have a vertical dipole with the negative surface component and a positive component
deeper in the brain. This type of phase reversal is seen in typical montages but not the referential
montage. A horizontal dipole has both a positive and negative surface component, which will cause
a phase reversal in the referential montage—a fairly unique feature to BECTS. Juvenile absence
epilepsy and juvenile myoclonic epilepsy have generalized spike and wave discharges, not focal
spikes as is present with BECTS. Autosomal-dominant frontal lobe epilepsy has seizures that arise
out of sleep, but spikes are seen frontally without a horizontal dipole.

Answer. a
92. A 63-year-old female who is on her seventh day of a hospital stay for a subarachnoid haemorrhage
(SAH acutely develops right arm weakness and slurred speech. Her transcranial Doppler demonstrates
increasing velocities, suggestive of cerebral vasospasm. Which of the following would be effective in
this scenario?
1. Hypervolemia
2. Hypertension
3. Balloon angioplasty
4. Haemodilution
5. Intraarterial beta-blockers

a. 1,2,3 are correct.

b. Only 2 is correct.

c. 1,2,3,4 are correct.

d. 1,2,3,4,5 are correct.

Solution. C.1,2,3,4 are correct.


For symptomatic vasospasm, although the evidence remains uncertain, standard therapy includes
“triple-H,” which includes hypertension, hypervolemia, and haemodilution. Angiography is also
used for symptomatic vasospasm, for deployment of balloon angioplasty, or for various
intraarterial drugs. These include calcium channel blockers (nimodipine, verapamil, nicardipine,
papaverine, amrinone, or milrinone. Notably, however, there is no evidence for, or standard
practice that includes, intraarterial beta -blocker injection

Answer. c

93. A 34-year-old woman presents with over 1 year of increasing headaches, followed by increasingly
irregular menstrual periods and lactation despite her nulliparous status. Her physical and neurological
examinations are normal. Magnetic resonance imaging demonstrates a 6-mm nonenhancing lesion
amid the normal structures of the Sella, without associated mass effect. Based on her most likely
diagnosis, which of the following is the best first-line treatment consideration?

a. Dopamine agonist

b. Chemotherapy

c. Stereotactic radiosurgery

d. Whole-brain radiation

Solution. A. Dopamine agonist


The woman most likely is suffering from a prolactin-secreting pituitary adenoma. Pituitary
adenomas are the third most common primary intracranial tumor, and prolactin is the most
commonly secreted hormone among these. A tumor generally <1 cm in size can be successfully be
treated with dopamine agonist. Chemotherapy is not generally used for these tumors. Stereotactic
radiosurgery may also be used for pituitary adenomas, although it is often reserved for recurrent
tumors after surgical resection due to associated risk of radiation exposure to the normal
surrounding pituitary,
as well as the optic pathways. Whole-brain radiation is rarely used for a single mass lesion, and
would cause excessive toxicity without clear benefit in this case.

Answer. a
94. A 45-year-old man presents to the emergency room with altered mental status. He has a history of
myasthenia gravis, but his immunosuppressant regimen is unknown. His wife states that he has poor
compliance with his medications. He has not been sick with infectious symptoms recently. On
examination, he is hypotensive and encephalopathic. He has central obesity with multiple bruises,
round facies, and striae on his abdomen, but the remainder of his neurological examination is normal.
Laboratory tests reveal hyponatremia, hyperkalaemia, and metabolic acidosis. What is the most
appropriate next best step to stabilize the patient?

a. Plasmapheresis

b. Intravenous immunoglobulins (IVIG

c. Elevate sodium.

d. Administer hydrocortisone.

Solution. D. Administer hydrocortisone.


The examination findings of central obesity, multiple bruises, striae, and round facies are
consistent with Cushing syndrome, likely related to chronic prednisone therapy for myasthenia
gravis. In this scenario, given that the patient has altered mental status and hypotension, one
should think about adrenal insufficiency and administer stress-dose steroids. The onset of adrenal
insufficiency in this case was likely prompted by abrupt withdrawal of steroids due to medication
noncompliance. Electrolyte abnormalities such as hyponatremia, hyperkalaemia, and metabolic
acidosis are also consistent with adrenal insufficiency. Plasmapheresis and IVIG are both options
to treat a myasthenic crisis, but the patient does not have symptoms of myasthenic crisis. A
correction of the electrolyte abnormalities is not the best answer in this case, as these will self-
correct with the administration of steroids.

Answer. d

95. A patient is diagnosed with constrictive pericarditis.All the following can be the cause except

a. Rheumatic fever

b. Radiation therapy

c. Tuberculosis

d. Hypothyroidism

Solution. (d Hypothyroidism. The causes of constrictive pericarditis are- rheumatic fever,


radiation therapy, tuberculosis, and purulent pericarditis.Hypothyroidism, however causes
pericardial effusion.

Answer. d
96. A 30 year old male presented with palpitations. The following ECG was recorded. What is the likely
diagnosis?

a. Sinus tachycardia

b. Atrial Fibrillation

c. PSVT

d. Ventricular tachycardia

Solution. b. Atrial Fibrillation. The ECG shows Narrow-qrs complex tachycardia with NO ‘p’
waves and Irregular R-R interval. The ECG also has a wavy baseline due to presence of ‘f’ wave
(fibrillatory wave which is specific for Atrial fibrillation.

Answer. b
97. A 50 year old male developed ascites.Fluid analysis done shows SAAG>1.1.Consider the following
diseases
A.Tuberculosis
B.Cirrhosis
C.CHF
D.Nephrotic syndrome
What can be the etiology

a. A,B,C

b. A,B,D

c. B,C

d. B,D

Solution. C. The analysis of ascites fluid is as follows

Answer. c
98. Consider the following statements about rheumatic fever
a. PR interval is increased in ECG
b. Chorea occurs in the absence of other manifestations after prolonged latent period
c. Mitral valve involvement is most common
d. Patient doesn’t need prophylaxis if no carditis is present
Which is true

a. a,b,c

b. b,c,d

c. a,b,d

d. a,c,d

Solution. A,
• Acute rheumatic fever (ARF is a multisystem disease resulting from an autoimmune reaction to
infection with group A streptococcus.
• Although many parts of the body may affected, almost all of the manifestations resolve
completely.
• The exception is cardiac valvular damage [rheumatic heart disease (RHD], which may persist
after the other features have disappeared.
• Up to 60% of patients with ARF progress to RHD.
• The endocardium, pericardium, or myocardium may be affected. Valvular damage is the
hallmark of rheumatic carditis.
• The mitral valve is most commonly affected.
• Myocardial inflammation may affect electrical conduction pathways, leading to P-R interval
prolongation (first degree AV block or rarely higher-level block and softening of the first heart
sound.
• Sydenham’s chorea commonly occurs in the absence of the other manifestations, follows a
prolonged latent prriod after group A streptococcal infection, and is found mainly in females.
• Chorea eventually resolves completely, usually within 6 weeks.
• Patient need prophylaxis even if no carditis.

Answer. a

99. A Patient has hepatitis B infection 2 years back from which recovered. What will be the serological
maker

a. Anti Hbc IgG

b. Anti Hbc IgM

c. HbsAG

d. Anti Hbe

Solution. A. Anti HbC IgG is marker of recovered infection.HbsAg will be negative.IgM anti Hbc
will be present during acute infection,

Answer. a
100. A patient admitted in ICU has ABG done which shows- serum Na 145 mmol/L,serum Cl- 100
mmol/L,serum HCO3- 14 mmo/L, pH-7.30 and paco2- 28 mmHg.What is the likely acid base disorder

a. Non anion gap metabolic acidosis

b. High anion gap metabolic acidosis

c. Respiratory acidosis

d. Respiratory alkalosis

Solution. B .pH and HCO3 are in same direction and decreased= metabolic acidosis.Anion gap is
Na-(Cl+HCO3= 140-114=26 which is high.

Answer. b

101. Assertion: Non-oliguric presentation of AKI is characteristic of Pre-renal disorders


Reason: Delay in treatment of Pre-renal disorders leads to Acute Tubular Necrosis

a. Both Assertion and Reason are independently true statements and the reason is the correct
explanation for the assertion

b. Both assertion and reason are independently correct statements but the reason is not the
correct explanation for the assertion

c. Assertion is independently a true statement, but the reason is independently a false statement.

d. Assertion is independently a false statement, but the reason is independently a true statement.

e. Both assertion and reason are false statements

Solution. d. Assertion is independently a false statement, but the reason is independently a true
statement.
Non-oliguric presentation of AKI is characteristic of Tubulo-Interstitial disorders. In Tubulo-
interstitial disorders the concentrating ability of the tubules are lost leading to dilute polyuria
with mild azotemia.
Delay in treatment of pre-renal i.e renal hypoperfusion leads to Acute tubular Necrosis.

Answer. d
102. The following feature/s favors the diagnosis of CKD
1. Loss of cortico-medullary differentiation
2. Shrunken kidneys
3. Isothenuria
4. Absence of Metabolic bone disease

a. 1,2,3

b. 1,3,4

c. 2,3

d. 1,4

Solution.

Answer. a

103. Which of the following is the most specific antibody for SLE?

a. Anti-Ro antibodies

b. Anti-Smith antibodies

c. Anti-dsDNA antibodies

d. Anti-histone antibodies

Solution. B. Anti-smith antibodies


The most specific antibody for SLE is the anti-Smith antibody; however, the sensitivity is only
30%-40%. Antinuclear antibody (ANA is used for screening and has a sensitivity of 95%. Anti-Ro
antibodies are present in 15% of patients with SLE and other connective tissue diseases. Anti-
dsDNA antibodies have a high specificity, but a sensitivity of only 70%. Drug-induced lupus ANA
antibodies are usually antihistone.

Answer. b
104. Match the following Clinical features with probable site of renal disorder:

a. Concentrated Oliguria p. Obstructive Uropathy

b. Dilute Polyuria q. Glomerulonephritis

c. Hematuria with hypertension r. Tubulo-Interstitial disorders

d. Loin pain s. Pre-renal disorders

Solution. c. 1-D, 2-C, 3-B, 4- A

Answer. a-s,b-r,c-q,d-p

105. 34-year-old female presented with frequent headaches and bodyache since the past 4-5 months.
She appears stressed and accepts that there is work pressure on her to meet deadlines. She also
complains of inability to focus on her work. She has disturbed sleep and wakes up tired on majority of
the days. Which antibody is most likely associated with her condition?

a. Anti-Jo

b. Anti- Ro

c. Anti- Sm

d. Anti-polymer

Solution. d. Anti-polymer antibody


The case described is most likely suffering from Fibromyalgia. She has a classical triad of aches
and pains > 3 months (Pain processing defect alongwith disturbed attention span (cognitive
defect and disturbed (NREM defect. Anti-polymer antibody is found in 50-60% of these
patients.

Answer. d
106. Peritoneal dialysis is associated with which of the following complication?

a. Peritonitis

b. Hypotension

c. Hypoglycemia

d. Sudden cardiac death

Solution. a. Peritonitis
Peritoneal dialysis needs insertion of intraperitoneal catheter. The risk of peritonitis is low albeit
present. Unlike Hemodialysis, the hemodynamic shift is low and hence risk of hypotension and
sudden cardiac death is not associated with PD. The high glucose content in the PD fluid results
in hyperglycemia and weight gain

Answer. a

107. A 30 year old female working as a shop keeper presents to the emergency department with
dyspnea, cough and chest tightness. She had such episode in the past year and required
hospitalisation. At present she is having breathing difficulty at rest , accessory muscle use and bilateral
wheeze on auscultation . Which of the following is usually not a complication of this disease ?

a. Type 2 Respiratory failure

b. ARDS

c. Respiratory arrest

d. Cyanosis

Solution. ARDS
The presentation is suggestive of acute severe exacerbation of asthma. It can progress to life
threatening form with severe hypoxemia (cyanosis, Respiratory acidosis (Type 2 respiratory
failure , respiratory muscle fatigue and later respiratory arrest . Asthma is usually not a
predisposing factor for ARDS

Answer. b
108. A 70-year-old man is referred because of 2-year history of progressive exertional dyspnea and an
abnormal lung examination. He has no significant cough, hemoptysis, or chest pain, and he has never
smoked. He worked as an roofer for 30 years and retired 10 years ago. On physical examination, fine
end-inspiratory crackles are auscultated at both lung bases. He has clubbing of the fingers. Chest
radiograph and pulmonary function tests are ordered. In addition to bilateral lower lobe reticular
interstitial opacities, what radiographic feature is likely to be found in this patient?

a. Eggshell calcification of lymph nodes

b. Upper lobe pulmonary nodules

c. Calcified pleural plaques

d. Cavitating pulmonary nodules and masses

Solution. C Calcified pleural plaques


The presentation is suggestive of Asbestosis . Asbestosis is generally associated with calcified
pleural plaques.

Answer. c

109. A Patient presented with high pH , low arterial CO2 and low plasma bicarbonate levels. Which of
the following conditions can cause such Acid base disorder?
i. Vomiting
ii. Hypoxemia
iii. Diuretic use
iv. Aspirin use
Select the right combination

a. i & ii are correct

b. iii & iv are correct

c. ii & iii are correct

d. ii & iv are correct

Solution. d. ii & iv are correct


High pH ,low arterial CO2 and low plasma bicarbonate levels is respiratory alkalosis usually due
to hyperventilation. Hypoxemia can cause hyperventilation , aspirin can stimulate respiratory
centre to cause hyperventilation. Vomiting and diuretic use cause metabolic alkalosis.

Answer. d
110. A 38-year-old man obtains a chest radiograph as part of a preemployment screening program.
After the radiograph reveals bilateral hilar lymphadenopathy but no parenchymal opacities, he is
referred to the pulmonary clinic where he reports no symptoms and has a normal physical examination.
He undergoes bronchoscopy with transbronchial biopsies, which reveal noncaseating granulomas.
Which of the following is true regarding this patient?
a. He will likely have an increased arterial PCO2 on arterial blood gas analysis.
b. He is not at risk for involvement in any other organ systems.
c. Elevated serum ACE levels and lymphopenia can be seen
d. Spontaneous remission is commonly seen with this stage of the disease.
Select the right combination

a. If a, b, c are correct

b. If a and c are correct

c. If c and d are correct

d. If all four (a, b, c, & d are correct

Solution. C
The Finding of noncaseating granulomas in a patient with bilateral hilar lymphadenopathy is
consistent with a diagnosis of sarcoidosis. Because he is asymptomatic with a normal
examination, he will likely have normal pulmonary function. Spontaneous remission is commonly
seen with his form of the disease. While he is at risk for involvement of other organs, he will not
necessarily develop pulmonary fibrosis without treatment. In the absence of severe lung disease,
he would be expected to have a normal arterial PCO2

Answer. c

111. A 78-year-old female is evaluated by her primary care physician for worsening dyspnea on
exertion. She has suffered several vertebral compression fractures with resulting kyphoscoliosis. Which
of the following findings suggest that her dyspnea may be related to Kyphoscoliosis ?

a. A total lung capacity of 55% predicted with FeV1/ FVC of 50%

b. FEV1/FVC ratio of 78%, with an FVC of 58% predicted.

c. A total lung capacity of normal limits with a normal FVC

d. FEV1/FVC ratio of 60 %, with near normal FVC

Solution. B
Kyphoscoliosis is typically characterized by a restrictive ventilatory defect on lung volume testing
with a total lung capacity (TLC less than 80% of predicted. Also Fev1 /FVC ratio can be normal to
increased with reduced FVC

Answer. b
112. A 34-year-old woman seeks evaluation for a complaint of cough and dyspnea on exertion that has
gradually worsened over 3 months. The patient has no past history of pulmonary complaints and has
never had asthma. She started working in a pet store approximately 6 months ago. Her duties there
include cleaning the reptile and bird cages. A high-resolution chest CT shows diffuse ground-glass
infiltrates in the lower lobes with the presence of centrilobular nodules. A transbronchial biopsy shows
an interstitial alveolar infiltrate of lymphocytes & plasma cells. There are also noncaseating
granulomas. All cultures are negative for bacterial, viral, and fungal pathogens. Which of the following
is the most likely diagnosis?

a. Aspergillosis

b. Hypersensitivity pneumonitis

c. Nonspecific interstitial pneumonitis related to collagen vascular disease

d. Sarcoidosis

Solution. B
The patient has a subacute presentation of hypersensitivity pneumonitis related to exposure to
bird droppings and feathers at work. Hypersensitivity pneumonitis is a delayed-type
hypersensitivity reaction that has a variety of presentations. Some people develop acute onset of
shortness of breath, fevers, chills, and dyspnea within 6–8 hours of antigen exposure. Others may
present subacutely with worsening dyspnea on exertion and dry cough over weeks to months.
Chronic hypersensitivity pneumonitis presents with more severe and persistent symptoms along
with clubbing. If a specific antigen is suspected, serum precipitins directed toward that antigen
may be demonstrated. However, these tests are neither sensitive nor specific for the presence of
disease. Chest radiography may be normal or show a diffuse reticulonodular infiltrate. Chest CT is
the imaging modality of choice and shows ground-glass infiltrates in the lower lobes.
Centrilobular infiltrates are often seen as well. Histopathologically, interstitial alveolar infiltrates
predominate, with a variety of lymphocytes, plasma cells, and occasionally eosinophils and
neutrophils seen. Loose, noncaseating granulomas are typical.

Answer. b
113. Type of Collagen deposited in the later phase of proliferative phase of wound healing?

a. 1,

b. 2,

c. 3,

d. 4

Solution. C , 3 Ref: Bailey & Love's Short practice of Surgery 27th edition Page The proliferative
phase lasts from the third day to the third week, consisting mainly of fibroblast activity with the
production of collagen and ground substance (glycosaminogly cans and proteoglycans, the growth
of new blood vessels as capillary loops (angioneogenesis and the reepithelialisation of the wound
surfa ce. Fibroblasts require vitamin C to produce collagen. The wound tissue formed in the early
part of this phase is called granulation tissue. In the latter part of this phase, there is an increase
in the tensile strength of the wound due to increased collag en, which is at first deposited in a
random fashion and consists of type Ill collagen. This proliferative phase with its increase of
collagen deposition is associated with wound contraction, which can considerably reduce the
surface area of a wound over th e first 3 weeks of healing.

Answer. c

114. Type of nerve degeneration at the proximal end of trauma to a nerve is

a. Wallarian degeneration

b. Transneuronal degeneration

c. Traumatic degeneration

d. All of the above

Solution. C.Traumatic degeneration Explanation: Ba iley & Love's SHORT PRACTICE of


SURGERY Ref :Bailey & Love's Short practice of Surgery, 27th edition Page no. 25 Distal to the
wound, Wallerian degeneration occurs. Proximally, the nerve suffers traumatic degeneration as
far as the last node of Ranvier. The regenerating nerve fires are attracted to their receptors by
neurotrophism, which is mediated by growth factors, hormones and other extracellular matrix
trophins. Nerve regeneration is characterised by profuse growth of new nerve fibres which sprout
fr om the cut proximal end. Overgrowth of these, coupled with poor approximation, may lead to
neuroma formation.

Answer. c
115. Most common site of the pressure sores?

a. Ischium

b. Sacrum

c. Greater trochanter

d. Occiput

Solution. Ans: A.Ischium


Explanation:
Bailey & Love's
SHORT PRACTICE of SURGERY
Ref :Bailey & Love's Short practice of Surgery,27th edition
Page no. 29
Pressure sores can be defined as tissue necrosis with ulceration due to prolonged pressure. Less
preferable terms are bed sores, pres
sure ulcers and decubitus ulcers.They should be regarded as preventable but occur in
approximately 5% of all hospital
ised patients (range 3-12% in published literat ure. There is a higher incidence in paraplegic
patients, in the elderly and in the sever
ely ill patient. Pressure sore frequency in descending order
Ischium
Greater trochanter
Sacrum
Heel
Malleolus (lateral then medial
Occiput

Answer. a
116. Cardiac tamponade causes which type of shock?

a. Hypovolemic shock

b. Cardiogenic shock

c. Obstructive shock

d. Distributive shock

Solution. C.Obstructive shock


Explanation:
Bailey & Lore; SHORT PRACTICE of SURGERY
Ref :Bailey & Love's Short practice of Surgery, 27th edition Page no. 13 In obstructive shock
there is a reduction in preload due to mechanical obstruction of cardiac filling. Common causes of
obstructive shock include cardiac tamponade, tension pneumothorax, massive pulmonary embolus
or air embolus. In each case, there is reduced filling of the left and/or right sides of the heart
leading to reduced preload and a fall in cardiac output. Cardiogenic shock is due to primary
failure of the heart to pump blood to the tissues. Causes of cardiogenic shock include myocardial
infarction, cardiac dysrhythmias, valvular heart disease, blunt myocardial injury
and cardiomyopathy. Cardiac insufficiency may also be due to myocardial depression caused by
endogenous factors (e.g. bacterial and humoral agents released in sepsis or exogenous factors,
such as pharmaceutical agents or drug abuse. Evidence of venous hyperte
nsion with pulmonary or systemic edema may coexist with the classical signs of shock.

Answer. c

117. All causes the shock except

a. Graves' disease

b. Hashimato's thyroiditis

c. Addison's disease

d. SIADH

Solution. D.SIADH
Explanation: Ref: Bailey & Love's Short pract ice of Surgery 27th edition Page 13 Endocrine
shock may present as a combination of hypovolaemic, cardiogenic or distributive shock. Causes of
endocrine shock include hypo and hyperthyroidism and adrenal insufficiency.
Hypothyroidism causes a shock state s imilar to that of neurogenic shock due to disordered
vascular and cardiac responsiveness to circulating catecholamines. Cardiac output falls due to
low inotropy and bradycardia. There may also be an associated cardiomyopathy.
Thyrotoxicosis may cause a hig houtput cardiac failure.Adrenal insufficiency leads to shock due
to hypovolaemia and a poor response to
circulating and exogenous catecholamines. Adrenal insufficiency may be due to preexisting
Addison's disease or be a relative insufficiency due to a pathological disease state, such as
systemic sepsis.

Answer. d
118. A 48 years old male was admitted with chief complaint of dysphagia since adulthood. The patient
had to drink a lot of water to help swallow solid or soft food. Complaint worsened in the last three mo
nths followed by odynophagia, nausea, and vomiting undigested, retained food. The patient also
complained about heartburn and was previously diagnosed with a variant of angina pectoris but his
symptoms didn't improve with medication. Oesophagogram showed d ilated distal esophagus with Bird
beak appearance. What is the diagnosis?

a. Barrett's Oesophagus

b. Plummer Vinson syndrome

c. Achalasia cardia

d. Nutcracker Oesophagus

Solution. C. Achalasia Cardia EXPLANATION: Ref: Bailey & Love's Short practice of Surge y,
27th edition, Page no.1095,1096,1097. The classic physiological abnormalities in Achalasia are a
non relax ing LOS and absent peristalsis in the body of the oesophagus. It is due to loss of the
ganglion cells in the myenteric (Auerbach's plexus, the ca use of which is unknown. The disease is
most common in middle life, but can occur at any age. It typically presents with dysphagia,
although pain (often mistaken for reflux is common in the early stages. Patients often present late
and, having had relativ ely mild symptoms, remain untreated for many years. Barium radiology
may show hold up in the distal esophagus, abnormal contractions in the esophageal body and a
tapering stricture in the distal esophagus, often described as a 'bird's beak appearance. beak
appearance.

Answer. c

119. Assertion(A: Menetrier Disease is characterized by large gastric rugal folds


Reason(R: Menetrier Disease is caused by the local expression of TGF a

a. Both (A and (R are correct and (R explains (A

b. Both (A and (R are correct but (R doesn't exp lain (A

c. (R is correct But (A is false

d. (A is correct But (R is false

Solution. A. Both (A and (R are correct and (R explains (A EXPLANATION: Ref: Schwartz's
Principles of Surgey 11th edition Page no.1154 Menetrier's disease is characteristically
associated with P rotein losing gastropathy and hypochlorhydria. There are large rugal folds in
the proximal stomach, and the antrum is usually spared. Mucosal biopsy shows diffuse
hyperplasia of the surface mucus secreting c ells and usually decreased parietal cells. It has
recently been suggested that M é n é trier's disease is caused by local overexpression of
transforming growth factor in the gastric mucosa, which stimulates the epidermal growth factor
receptor, a receptor tyr osine kinase, on gastric SECs. This results in the selective expansion of
surface mucous cells in the gastric body and fundus.

Answer. a
120. A 35yr old female came to the OPD with the chief complaints of Right upper quadrant pain and
ascites. She has the history of Oral contraceptive pills intake. On blood workup, there is a elevated
platelet and WBC level. On Ultrasound, Caudate love hypertrophy is seen. What is the most possible
diagnosis?

a. Budd Chiari syndrome

b. Pyogenic liver disease

c. Liver cirrhosis

d. Hepatic adenoma

Solution. A. Budd Chiari syndrome EXPLANATION: Ref: Schwartz's Principles of Surgery 11th
edition Page no.1368 1369 Budd Chiari syndrome (BCS is an uncommon congestive hepatopathy
characterized by the obstruction of hepatic venous outflow. BC S is defined as primary when the
obstructive process involves an endoluminal venous thrombosis. essential thrombocythemia or
polycythemia rubra, account for approximately 35% to 50% of the primary cases of BC.
Anticardiolipin antibodies, hyperhomocysteinem ia and oral contraceptive use all have been
shown to be risk factors for BCS. Caudate lobe hypertrophy occurs in approximately 50% of cases
and is due to the fact that the caudate lobe has direct venous drainage into the IVC.

Answer. a

121. The following ERCP show?

a. Normal cholangiogram

b. Bile duct stricture

c. Common Bile duct obstruction

d. Gall stones

Solution. A.Normal Cholangiogram Explanation: Ref: Bailey & Love's Short practice of Surgery
27th edition Page no.1193
It is a Normal Cholangiogram with no stricture and gall stones.There are no signs of distal bile
duct obstruction.

Answer. a
122. The following image on plain xray abdomen shows a condition is a predisposition for ?

a. Choledochal cyst

b. Chronic cholecystitis

c. Empyema of gall bladder

d. Cancer of gall bladder

Solution. D.cancer of gallbladder Explanation: Ref :Bailey & Love's Short practice of Surgery,
27th edition page no: 1190 & 1210
It is a plain radiographic picture of porcelain gallbladder. Calcification of The gallbladder wall,
pres umably due to chronic inflammation (porcelain gallbladder, is also associated with a small
increased risk of cancer. The exact cause is not known. But gall stones are a cause for porcelain
bladder. This is more commonly seen on computed tomography(ct.

Answer. d
123. The following tumor shows which type of klatskin Tumour as per bismuth corlette classification ?

a. TYPE 1

b. TYPE 2

c. TYPE 3A

d. TYPE 3B

Solution. B, TYPE 2

Answer. b
124. In strasberg classification Lateral Duct injury is

a. Type A

b. Type B

c. Type C

d. Type D

Solution. Type D

Answer. d

125. A 35yr old male patient who is admitted for headache and ataxia is found to have a hemangio
blastoma On ultrasound, there is evidence for renal cell carcinoma and he is found to have a pancreatic
abnormality. What is that pancreatic co ndition?

a. Pancreatic divisum

b. Annular pancreas

c. Ectopic pancreas

d. Congenital cyst of Pancreas.

Solution. D. Congenital cyst of pancreas EXPLANATION: Ref :Bailey & Love's Short practice of
Surgey, 27th edition Page no.1220 and 1418 This is von Hippel Lindau syndrome Gene affected
VHL (3p25 26 Protein affected pVHL Age of incidence 25 45
Associated conditions Haemangioblastoma ( Which causes head ache and ataxia Pancreatic
neuroendocrine tumours Pheochromocytoma Renal cysts Pancreati c cysts Renal cell carcinoma
Ovary cystadenoma Epididymalcystadenomame.

Answer. d
126. Assertion(A: Appendicitis is caused by the Bacterial infection alone
Reason(R: The blind feature of the appendix lumen is responsible for the acute appendicitis.

a. Both (A and (R are correct and (R explains (A

b. Both (A and (R are incorrect

c. (R is correct But (A is false

d. (A is correct But (R is false

Solution. B. Both (A and (R are incorrect EXPLANATION : Ref :Bailey & Love's Short practice of
Surgery, 27th edition Page no.1302 appendicitis is clearly associated with bacterial proliferation
within the appendix, no single organism is responsible. A mixed growth of aerobic and anaerobic
organisms is usual. The initiating event causing bacterial proliferation is controversial.
Obstruction of the appendix lumen has been widely held to be important, and some form of
luminal obstruction, either by a faecolith or a stricture, is found in the majority of cases.
Lymphoid hyperplasia narrows the lumen of the appendix, leading to luminal obstruction. Once
obstruction occurs, continued mucus secretion and inflammatory exudation increase intraluminal
pressure, obstructing lymphatic drainage. edema and mucosal ulceration develop with bacterial
translocation to the submucos a. Resolution may occur at this point either spontaneously or in
response to antibiotic therapy.

Answer. b

127. Treatment of choice for the intraperitoneal rupture of rectum?

a. Suture repair and defunctioning with stoma

b. Hartmann's procedure

c. Rectal drainage, Washout and defunctioning colostomy

d. Proctosigmoidectomy

Solution. A. Suture repair and defunctioning with stoma Explanation: Ref: Bailey & Love's Short
practice of Surgery 27th edition Page no 1321 The rectum is examined under general anaesthetic
with a finge r and a sigmoidoscope. If penetrating injury is confirmed, laparotomy or laparoscopy
is required. If an intraperitoneal rupture of the rectum is found, the perforation is closed with
sutures and the rectum defunctioned with a stoma. In the event that the rectal injury cannot be
repaired, a Hartmann's procedure may need to be performed. If the rectal injury is below the
peritoneal reflection, wide drainage from below is indicated, with rectal washout and a
defunctioning colostomy. Care must be taken to preserve sphincter function during debridement
of the perineal wounds. Antibiotic cover must be provided against both aerobic and anaerobic
organisms.

Answer. a
128. The anorectal ring is formed by all except

a. Puborectalis

b. Middle part of internal sphincter

c. Deep part of external sphincter

d. Conjoined longitudinal muscle

Solution. B.Middle part of internal sphincter EXPLANATION:

Ref :Bailey & Love's Short practice of Surgery,27th edition Page no. 1339 The anal canal
commences at the level where the rectum passes through the pelvic diaphragm and ends at the
anal verge. The muscular junction between the rectum and anal canal can be felt with the finger
as a thickened ridge the anorectal 'bundle' or 'ring'. The anorectal ring T he anorectal ring marks
the junction between the rectum and the anal canal. It is formed by the joining of the puborectalis
muscle, the deep external sphincter, conjoined longitudinal muscle and the highest part of the
internal sphincter. The anorectal ring can be clearly felt digitally, especially on its posterior and
lateral aspects.

Answer. b
129. The following sign is seen in raised ICT compressing

a. Dorsal midbrain

b. Ventral midbrain

c. Cerebellum

d. Pons

Solution. A.Dorsal midbrain EXPLANATION: Ref :Bailey & L ove's Short practice of Surgery,27th
edition Page no. 652 In infants, the fontanelle is tense and bulging, with an increase in head
circumference and bulging scalp veins. As pressure rises, conscious level is impaired. In children
Pari naud's syndrome re sults from dorsal midbrain compression, with a loss of upgaze known as
sunsetting. Raised IP requires urgent evaluation and management: delay risks progression to
cerebral herniation (see Chapter 24 resulting in cardiovascular instability, neurological de ficit
and death. Vision may also deteriorate rapidly and irreversibly.

Answer. a

130. Assertion(A: Skin graft can't be used on the cartilage


Reason(R: Granulation tissue is essential for the success of the skin graft.

a. Both (A and (R are correct and (R explains (A

b. Both (A and (R are correct but (R doesn't explain (A

c. (R is correct But (A is false

d. (A is correct But (R is false

Solution. Solution. Ans:A.Both (A and (R are correct and (R explains (A EXPLANATION: Ref
:Bailey & Love's Short practice of Surgery, 27th edition Page no. 635 Split thickness skin grafts
survive initially by imbibition of plasma from the wound bed; after 48 hours, fine anastomotic
connections are made, which lead to inoculation of blood. Capillary in growth then completes the
healing process with f fibroblast maturation. Because only tissues that produce granulation will
support a graft, it is usually contraindicated to use grafts to cover exposed tendons, cartilage or
cortical bone. Skin grafts inevit ably contract, with the extent of contracture determined by the
amount of dermis taken with the graft and the level of postoperative splintage and physiotherapy
applied to the grafted site.

Answer. a
131. A 50yr old man presents with the oral cavity mass. It is susp ected to be a salivary gland tumor.
An FNA Biopsy was done and it was inconclusive. What is next step?

a. Open biopsy

b. Repeat FAB with ultrasound

c. MRI

d. Incisional biopsy

Solution. B.Repeat FNAB with ultrasound EXPLANATION: Ref: Schwartz's Principles of Surgery
11th edition Page no.650 Following a thorough history and physical examination, an FNA biopsy
should be performed to provide an accurate preoperative diagnosis in 70% to 80% of cases when
reviewed by an experienced cytopathologist. If the biopsy is nondiagnostic, a repeat biopsy should
be performed under imageguidance, typically with an ultrasound. An open or incisional biopsy
should be avoided because of the risk of tumor spillage and cutaneous spread. Also, this approach
is fraught with risk to th e facial nerve. Salivary gland tumors are worked up with appropriate
imaging, typically with an MRI because of the increased soft tissue definition. FNA and imaging
results are critical in guiding the surgeon to the extent of surgery.

Answer. b

132. Assertion(A: Witch's milk is produced by the fullterm babies at 4th day of life.
Reason(R: It is due to the increased prolactin in fetal blood in response to drop in maternal estrogen .

a. Both (A and (R are correct and (R explains (A

b. Both (A and (R are correct but (R doesn't explain (A

c. (R is correct But (A is false

d. (A is correct But (R is false

Solution. Explanation: Ref :Bailey & Love's Short practice of Surgery, 27th edition Page no. 865
Mastitis of infants is at least as common in boys as in girls. On the third or fourth day of life, if the
breast of an infant is pressed lightly, a drop of colourless fluid can be expressed; a few days later,
there is often a slight milky secretion, which disappears during the third week. This is popularly
known as 'witch's milk' and is seen only in full term infants. It is caused by stimulation of the fetal
breast by prolactin in response to the drop in maternal estrogens and is essentially physiological.
True mastitis is uncommon and is predominately cause d by Staphylococcus aureus.

Answer. a
133. Investigation of Choice for differentiating follicular adenoma and follicular carcinoma?

a. Histological examination

b. FNAC

c. Trucut biopy

d. CT

Solution. A : Histological Examination EXPLANATION: Ref :Bailey & Love's Short practic e of
Surgery, 27th edition Page no. 813 Follicular adenomas present as clinically solitary nodules
(Figure 50.21 and the distinction between a follicular carcinoma and an adenoma can only be
made by histological examination; in the adenoma there is no i nvasion of the capsule or of
pericapsular blood vessels. For this reason, FNA, which provides cytologic detail but not tissue
architecture, cannot differentiate between benign and malignant follicular lesions. Diagnosis and
treatment is therefore, by wide excision, i.e. total lobectomy: The remaining thyroid tissue is
normal so that prolonged follow up is unnecessary.

Answer. a

134. The most common clinical manifestation of Hyperparathyroidism?

a. Nephrolithiasis

b. Osteopenia

c. Osteoporosis

d. Pancreatitis

Solution. A.Nephrolithiasis EXPLANATION: Ref :Bailey & Love's Short practice of Surgery,27th
edition Page no. 825 Most patients will, however, have some vague constitutional symptoms, such
as fatigue, muscle weakness, depression or some mild memory impairment on q uestioning. The
presence of kidney stones remains the most common clinical manifestation of symptomatic PHPT.
Between 15% and 20% of patients will have nephrolithiasis and over 40% of patients will have
hypercalciuria. Increasingly, postmenopausal women pr esent with significant osteopenia or
osteoporosis in the distal one third of the radius with a minimal reduction in the lumbar spine,
which prompts further investigation. PPT may present with pancreatitis, although it is rarely seen
in patients with milder forms of the disease.

Answer. a
135. Which of the following is not true about medullary carcinoma?

a. Can occur without MEN 2

b. Elevated Carcinoembryonic antigen

c. Diarrhea will be present

d. Take up radioactive iodine

Solution. D.Take up Radioactive iodine Explanatio n: Ref :Bailey & Love's Short practice of
Surgery, 27th edition Page no. 820 21 High levels of serum calcitonin and carcinoembryonic
antigen are produced by many medullary tumours, which should be tested for in suspected cases.
Diarrhoea is a feature in 30% of cases and this may be due to 5 hydroxytryptamine or
prostaglandins produced by the tumour cells. Medullary carcinoma may occur in combination
with adrenal phaeochromocytoma and hyperparathyroidism (HPT (usually due to hyperplasia in
the syndrome known as multiple endocrine neoplasia type 2A (MEN 2A. The familial form of the
disease frequently affects children and young adults, whereas the sporadic cases occur at any age
with no sex predominance. tumours are not TSH dependent and d o not take up radioactive
iodine.

Answer. d

136. A man presented with fractures of 4 th to 10 th ribs and respiratory distress after RTI. He is
diagnosed of flail chest and a PaO2 of & lt;60%. Management is

a. Tracheostomy

b. IPPV with oral intubation

c. Fixation of ribs

d. Strapping of chest

Solution. In cases of flail chest if is having respiratory distress like in this case the treatment is
ETT and IPPV(intermittent positive pressure ventilation. If patient doesn’t have respiratory
distress than treatment is oxy gen by mask and analgesia. Ref ATLS 2018(10 TH Edition

Answer. b

137. Obesity is a risk factor

a. True

b. False

Answer. a
138. Previous PPH is a risk factor

a. True

b. False

Answer. a

139. Golden hour of PPH starts after full cervical dilatation

a. True

b. False

Answer. b

140. Uterine massage is first step in management

a. True

b. False

Answer. a

141. B-lunch is used if medical management fails

a. True

b. False

Answer. a
142. 40 year old patient complains of grayish discharge. Microscopic finding is given below

1. Group of bacteria adherent to epithelial cells


2. Nugent score on gram stain used for diagnosis
3. Drug of choice in pregnancy is metronidazole
4. Associated with garderella vaginalis, peptostreptococcus and bacteroides

a. If 1and 2 only are true

b. If 3 and 4 only are true

c. If 1,2 and 3 only are true

d. If 1,2,3 and 4 are true

Solution. D: If 1,2,3 and 4 are true

Answer. d
143. Assertion: to define anaemia in pregnancy lower cut off of Hb is taken than non pregnant
scenario
Reason: physiological hemodilution is seen in pregnancy

a. Both assertion and reason are true statements and reason is correct explanation of assertion

b. Both reason and assertion are true statements but reason is not correct explanation of
assertion

c. Assertion is true but reason is false

d. Assertion is false reason is true

e. Both are false

Solution. A: Both assertion and reason are true statements and reason is correct explanation of
assertion

Answer. a

144. Regarding MgSO4 not true is

a. 4gm 20% i/v is given @of 1gm/min

b. Give i/m in each buttock

c. Mgso4 can treat hypertension

d. I/V loading dose is 8mlMgSO4 and 12 ml 0.9% NS

Solution. C: Mgso4 can treat hypertension

Answer. c

145. All are used in the treatment of PCOS except

a. Clomiphene citrate

b. Ulipristal

c. Laparoscopic ovarian drilling

d. Letrozole

Solution. B: Ulipristal

Answer. b
146. 38 year old lady presented to gynae OPD with cytology report showing AGUS, next step will be
1. Repeat PAPS
2. Endocervical curettage
3. Colposcopy
4. Endometrial curettage
Answer using the following code

a. If 1 and 2 are true

b. If 1,2 and 3 are true

c. If 2,3 and 4 are true

d. If 2 and 3 are true

Solution. C: If 2,3 and 4 are true

Answer. c

147. ESR in pregnancy increase because of

a. Fibrinogen

b. Albumin

c. Antithrombin

d. Platelets

Solution. A: Fibrinogen

Answer. a
148. A 25-year-old G2P1 woman presents at 9 weeks’ gestation for routine prenatal care She is
interested in first-trimester screening. Which of the following tests will your order for her first
trimester screening of downs
1. HCG
2. AFP
3. PAPPA
4. NT
Answer using the code below

a. 1,2 and 3

b. 1,3 and 4

c. 1,2,3 and 4

d. 1 &3

Solution. B: 1,3 and 4


First-trimester screening for Down syndrome has become the standard of care, though not all
women can get access to the testing because of the lack of availability of trained sonologists to
provide NT ultrasound and variability in the insurance coverage for first-
trimester testing. The serum screen is composed of PAPP-A and free β-hCG and has a 60%
sensitivity for Down syndrome alone. NT alone has a 70% sensitivity and when added to the
serum screen, the two together have an 80% sensitivity. If the NT is increased and there is no
aneuploidy, the fetus is at increased risk, in particular, for cardiac anomalies. The sequential
screen is a test that combines the first-trimester and second-trimester tests. This combination
gets to a 90% to 95% sensitivity, so it has a greater sensitivity than first-trimester screening
alone.

Answer. b
149. Match the following ovarian tumours with the characteristic features

a. LDH and HCG p. brenners tumour

b. wallthard cell nest q. dysgerminoma

c. signet ring cells r. endodermal sinus tumour

d. AFP and LDH s. krukenbergs

Solution. ANSWER. B: 1b 2a 3d 4c
The main tumour marker of
dysgerminoma is LDH although HCG
and PLAP may also be positive.
Brenners tumour shows wallthard cell
nest. It is a epithelial ovarian tumour.
The main tumour marker for
endodermal sinus tumour is AFP
although LDH can also be raised. It
shows schiller duval bodies on hpe.
Signet ring cells are characteristic of
krukenbergs tumour which is
metastatic ovarian tumour from gastric
cancer

Answer. a-q,b-p,c-s,d-r

150. An 18-year-old young woman presents to your office with a complaint of amenorrhea On physical
examination, you note normal appearing axillary and pubic hair. Her breast development is normal.
Pelvic examination reveals normal appearing external genitalia, and a shortened vagina ending in a
blind pouch. Which of the following tests would be your first step in determining the diagnosis?

a. Karyotype

b. Pelvic ultrasound

c. Serum follicle-stimulating hormone (FSH

d. Diagnostic laparoscopy

Solution. B: Pelvic ultrasound


A pelvic ultrasound should be the first step, aimed to determine the presence or absence of the
uterus, tubes, and ovaries. Given the presence of normal telarche (breast development appears
normal and adrenarche (pubic hair is present, a random FSH would not be helpful. A karyotype
and diagnostic laparoscopy may also be helpful in determining a final diagnosis, but an ultrasound
should be the next step

Answer. b
151. Consider the following statements regarding non stress test
1. Reactive NST indicates healthy fetus
2. NST is an observed association of fetal breathing with fetal movements
3. NST has a low false negative rate
4. Testing should be started at 20 weeks
Which of the statements given above is/are correct

a. 1 and 3

b. 2 only

c. 3 only

d. 1 and 4

Solution. A: 1 and 3
NST is a reliable technique for evaluarion of fetal status. As a rule reactive NST as opposed to non
reactive NST has been associated with good pregnancy outcome. Negative NST means its recative
. false negative NST is uncommon and it has low false negative rate of 1%
Non reactive test is considered positive. The false positive rate is high 75-90% and therefore a
positive test must be followed up with BPP. NON REACTIVE TEST has low sensitivity and positive
predictive rate of 28%

Answer. a
152. Which of these instruments would you use during CS

Answer using the code given below


a. 1,2,3 and 4

b. 1,3 and 4

c. 1,3, 4 and 5

d. 1 and 4

Solution. B: 1,3 and 4

Answer. b
153. Match the congenital anomalies with their correct diagnosis
a.

p. Omphalocele

q. Anencephaly

b.
c.

r. Duodenal atresia

d.

s. Spina bifida

Solution. ANSWER. B: A2 B1 C4 D3

Answer. a-q,b-p,c-s,d-r
154. What is the most common karyotype associated with the condition shown below

a. 46XX

b. 46XY

c. 69 XXY

d. 69 XYY

Solution. A: 46XX
This is the snowstorm appearance characteristic to complete mole. The most common karyotype
associated with it is 46XX with all chromosomes of paternal origin. About 10% can have karyotype
46XY which is thought to occur by fertilization of empty pvum with 2 sperms

Answer. a
155. Which of the following are included in criteria to make a diagnosis of severe preeclampsia
1. BP>=160
2. oliguria
3. Serum creatinine > 1.1
4. Headache, scotomas
Answer using the code given below

a. 1 &3

b. 2 &3

c. 1,3&4

d. 2,3& 4

Solution. C: 1,3&4
The criteria for making a diagnosis of preeclampsia with severe features are
a SBP>=160 and/or DBP >=110mmhg
b New onset cerebral or visual disturbance like headcache, photopsia scotomas blindness
c Liver enzymes raised to more than 2 times the upper limit of normal
d Platelet count < 1 lakh
e Serum creatinine >1.1
f Pulmonary edema
IUGR, oliguria and degree of proteinuria are no longer defining criteria for PE with severe
features

Answer. c

156. Arrange the investigations/ procedures based on the POG at which they are performed from
earliest to late
1. OGTT
2. Level 2 usg
3. NT scan
4. Amniocentesis
Answer using the code given below

a. 3241

b. 3421

c. 3214

d. 1324

Solution. B: 3421
NT scan done at 11-13 weeks, amniocenetsis done at 15-18 weeks, level 2 usg done between 18-
22 weeks and OGTT done beyond 24 weeks

Answer. b
157. Regarding HPV vaccine not true is

a. Contraindicated in pregnancy

b. Made from E1 protein

c. Ideal age 11-12 yrs

d. Effective against vulvar, vaginal, penile and anal cancers

Solution. B: Made from E1 protein


Ideal age for HPV vaccine is between 11-12 year. It can be given between 9-26 years of age
Women who are at high risk can receive the vaccine upto 45 years of age
The nonavalent vaccine is made of the following strains
6, 11, 16, 18, 31, 33, 45, 52, 58
It is protective against oral, anal, penile, nasopharyngeal vaginal and vulvar cancers as well

Answer. b

158. A 36 year old woman presents with inability to conceive for last 3 years, she has had one child 4
years back which was a spontaneous conception. On evaluation she has 5cm intramural fibroid what is
the subsequent course of action for her fibroid management

a. Expectant management

b. UAE

c. Myomectomy

d. GnRH agonist

Solution. A: Expectant management


There is no role of empirical removal of asymptomatic or minimally symptomatic fibroids in
patients before attempting pregnancy to improve fertility because risk of new recurrent fibroids
that additional intervention as they are pursuing pregnancy. For patients who desire pregnancy
and presenr with bulk symptoms due to intramural or subserosal fibroid myomectomy either
abdominal or laparoscopic is recommended. If the fibroid is asymptomatic but is cornual fibroid
blocking the tube or submucosal fibroids causing abortions then myomectomy is recommended

Answer. a
159. Match the clinical conditions with their correct hormone profile

a. PCOS p. Normal LH normal FSH normal estradiol

b. Ashermans syndrome q. High LH high FSH low estradiol

c. Kallmann syndrome r. High LH normal FSH normal estradiol

d. Turners syndrome s. Low LH low FSH low estradiol

Solution. B: 1c 2a 3d 4b
PCOS has persistently high LH levels
normal FSH and normal estradiol levels
Turners is gonadal dysgenesis and
causes hypergonadotropic
hypogonadism hence high LH FSH and
low estradiol
Kallmann causes hypogonadotropic
hypogonadism hence low levels of LH
FSH and estradiol
Ashermans is a defect at the level of
end organ and HPO axis is working
normally so all hormone levels are
normal

Answer. a-r,b-p,c-s,d-q

160. Which of the following clinical findings is seen in women with ashermans syndrome
1. Hypomenorrhea
2. Primary amenorrhea
3. Cyclic pain
4. Recurrent pregnancy loss
Answer using the code given below

a. 1,2 &3

b. 1 &3

c. 1& 4

d. 1,3& 4

Solution. D: 1,3& 4
Recurrent pregnancy loss occur in ashermans due to abnormalities of implantation or due to
insufficient vascualrization. Cyclical pain occurs due to obstruction of menstrual flow and /or
hematometra. Infertility is reported in 7-40% patients. Abnormal uterine bleeding
is seen in 70-95% of owmne with asherman syndrome. Most common is secondary amenorrhea
followed by hypomenorrhea.

Answer. d
161. Arrange the following causes of neonatal mortality across the world from high to low incidence:
i prematurity;
ii sepsis;
iii congenital causes;
iv intrapartum events

a. i,ii,iii,iv

b. i,iv,ii,iii

c. i,iv,iii,ii

d. i,iii, iv, ii

Solution.

Answer. b
162. True about CMV infection?

a. Triad of SNHL, enamel hypoplasia and calcification in 20-30 %

b. Mothers with IgG-CMV have low risk of transmitting disease

c. 30-40% asymptomatic CMV have high risk of SNHL later

d. Only 20-30% CMV infection is symptomatic

Solution. b Mothers with secondary CMV infections have less chance of transmission of CMV to
their babies.

Answer. b

163. A newborn presents with purulent conjunctivitis and pneumonia. Baby has no fever and has
normal activity. On the gram stain of the purulent discharge, there is no organism found. What is your
diagnosis?

a. Neisseria gonorrhoea

b. Pseudomonas

c. Chlamydia trachomatis

d. Mycoplasma

Solution. c Chlamydia
Ophthalmia neonatarum
- <4 wk of age most common eye disease of newborns.
- Chlamydia trachomatis is the most common

Answer. c

164. Hypokalemic, hypercholemic metabolic alkalosis with hypertension. Diagnosis

a. Bartter syndrome

b. Gitelman syndrome

c. Liddle syndrome

d. Gordon syndrome

Solution. c Liddle syndrome


ENaC function is increased in Liddle's syndrome
- mineralocorticoid excess, such as hypertension, hypokalemia and metabolic alkalosis.
- Presentation at a young age,
- plasma and urinary aldosterone levels are reduced, not increased as in primary aldosteronism.

Answer. c
165. ACTH is the drug of choice for

a. Tuberous sclerosis

b. West syndrome

c. Rolandic epilepsy

d. JME

Solution. b West syndrome


Also remember for infantile spasms in tuberous sclerosis, the drug of choice is Vigabatrin

Answer. b

166. Congenital absence of seminal vesicles and vas deferens is seen in

a. Cystic fibrosis

b. Klinefelter syndrome

c. Kallman syndrome

d. Chromosomal microdeletion

Solution. a Cystic Fibrosis has CBVAD [congenital bilateral absence of vas deferens and
epididymis]

Answer. a

167. Tongue fasciculations are seen in

a. Myasthenia gravis

b. Spinal Muscular Atrophy

c. Duchenne muscular dystrophy

d. GBS

Solution. b SMA has lower motor neuron weakness, so has fasciculations

Answer. b
168. You see a 1-year-old female whose problems began in infancy. Initially, she developed hypotonia.
Now she has progressed to the point where she has severe weakness. She has hyporeflexia on
examination and muscle fasciculations are noted—particularly of her tongue. She has not had any
seizures or fever.
Which of the following would not be of any use in treatment of previous case?

a. steroids

b. rispiplam

c. onasemnogene abeparvovec-xioi

d. Zolgensma

e. Nusinersen

Solution. A steroids have no use in SMA

Answer. a

169. 40 year old male got his routine blood workup done in which his ALP Levels came out to be 8-10
times the normal values. He told about the history of occasional backpain as well. Which of the
following statements ain't true about this condition?

a. This is likely to be associated with paramyxoviral infection

b. Salmon Calcitonin will be helpful in alleviating the bone pains

c. Bisphosphonates are the drug of choice

d. This is a low turnover bone disease

Solution. D
This is a clinical spectrum of Paget's disease. There is excess osteoblastic bone formation and
excess osteoclastic resorption leading to a high turnover bone disease.

Answer. d
170. A 10 year old child is brought in with swelling over both the mandibles. There is a history of over
3 months. As such there is no pain or fever now. The mother gives history of dental infection very
recently. X-ray shows laminated periosteal reaction over both the sides. Which of the following
statements is true about the condition above?

a. There is no pus formed usually in this condition

b. It is due to an excess periosteal reaction by an extremely sensitive periosteum

c. It represents a subacute form of osteomyelitis

d. Most of the cases respond well to conservative management

Solution. C
This is a case of Garres OM which represents a long term chronic form of osteomyelitis

Answer. c

171. A 7 year old boy is brought by parents to Ortho OPD with chief complains of dull aching pain and
inability to squat. His problems started with intense pain in the right hip two weeks ago for which he
was treated conservatively and symptomatically. His blood investigations show raised ESR with other
counts almost normal. X-rays were done which showed osteopenia and mild erosions around the hip
joint mainly towards the acetabular side. Which of the following is the correct diagnosis?

a. Perthes' disease

b. Septic Arthritis

c. Tuberculosis hip

d. Slipped Capital Femoral Epiphysis

Solution. C: Tuberculosis hip


His problems started with intense pain in the right hip two weeks ago for which he was treated
conservatively and symptomatically: Stage I TB hip
X-rays were done which showed osteopenia and mild erosions around the hip joint mainly towards
the acetabular side : Stage II /III TB Hip
Had it been Perthes', X rays would have shown destruction ofhead of femur rather than
acetabular involvement.

Answer. c
172. A 42 year old female presents with tingling and numbness in right hand that awakens her in the
middle of the night for last two weeks. She also gives history of being treated for diabetes and
hypothyroidism for last 3 years by an endocrinologist. Which of the following ain’t true about this
condition from the following options ?

a. Splintage is helpful only in few initial months of onset of symptoms

b. Motor Symptoms appear early in the course of the disease

c. Durakn’s Test is the most specific clinical test in the diagnosis

d. It is the most common focal compression neuropathy

Solution. B
Motor symptoms are the last to appear

Answer. b

173. A 26 year old male has been complaining of back pain for last one year. His pain radiates from the
back to right buttock and thigh down to legs. On examination he has mild sensory loss over dorsum of
foot and weakness of extension of great toe on the right side. Which of the following is the most
probable diagnosis ?

a. PIVD L4-L5 Postero lateral / Paracentral

b. PIVD L5-S1 Postero central

c. PIVD L4-L5 Far Lateral / Foraminal

d. PIVD L5-S1 Postero lateral / Paracentral

Solution. A
Dermatome and myotome explained in the question means L5 So it will be either Posterolateral /
Paracentral L4-L5 Disc Prolapse or Far Lateral/ Foraminal L5 -S1 Disc Prolapse or both

Answer. a
174. What is the staging of following cancer ?

a. T1 N1 M0

b. T2 N1 M0

c. T3 N0 M0

d. T4a N1 M0

Solution. d As there is laryngeal cartilage invasion it is classified as T4a. Also note for future
exams MRI is IOC for laryngeal cartilage invasion.

Answer. d
175. 34 year old woman was brought to the AIIMS Emergency after road traffic accident with GCS-3.
She was intubated and NCCT was advised and was found to be normal with no space occupying lesion.
What is the most likely diagnosis?

a. Cerebral contusion

b. Diffuse axonal injury

c. SDH

d. EDH

Solution. b DAI
Diffuse axonal injury (DAI, also known as traumatic axonal injury (TAI, is a severe form of
traumatic brain injury due to shearing forces. It is a potentially difficult diagnosis to make on
imaging alone, especially on CT as the finding can be subtle, however, it has the potential to
result in severe neurological impairment. The diagnosis is best made on MRI where it is
characterized by several small regions of susceptibility artifact at the grey-white matter junction,
in the corpus callosum, and in more severe cases in the brainstem, surrounded by FLAIR
hyperintensity.

Answer. b
176. What is the least useful investigation the diagnosis in the patient with following barium image?

a. Manometry

b. UGI Endoscopy

c. Timed barium swallow

d. 24 hour PH monitoring

Solution. d
Barium swallow shows Achalasia cardia and rest all are useful tests for the condition

Answer. d
177. Which of the following are indications for radioiodine treatment?
1 Relapsed Graves’ disease
2 Thyrotoxicosis in young children
3 Multinodular goitre
4 Severe ophthalmopathy
5 Pregnancy and lactation.

a. 1 +3 +4

b. 1+3

c. 1+2+3+4

d. 1+2+3+4+5

Solution. B. 1+3
Radioiodine treatment recommends the administration of enough radioiodine to achieve
normalthyroid status. Hypothyroidism occurs in 15–20 per cent at 2 years. It is contraindicated in
young children, pregnant and lactating mothers. It is the definitive choice of treatment in cases of
relapsed Graves’ disease and in multinodular goitre and adenomas. One cause for concern is that
radioactive iodine therapy has been associated with worsening Graves' ophthalmopathy.
Radioiodine treatment carries a substantial risk of exacerbating pre-existing thyroid eye disease it
should be avoided as far as possible in patients with active or severe ophthalmopathy.

Answer. b
178. Identify the MRI sequence shown to you?

a. T1WI

b. T2WI

c. FLAIR

d. SWI

Solution. d SWI

Answer. d
179. A 20 year old man comes to you with alopecia. He reports that the patches are coming since the
past 3 months. On examination, his palm and sole are showing scaly lesions. He also reports perianal
lesions. The diagnosis for this condition is

a. Syphilis

b. Psoriasis

c. Alopecia areata

d. Androgenetic alopecia

Solution. a
The image refers to a condition seen in secondary syphilis called moth eaten alopecia. The clues
in the question are scaling on the palm and the sole which is the rash of secondary syphilis and
perianal lesions which may refer to a condition called as condyloma lata. Psoriasis has scaling but
does not have alopecia. Alopecia areata will not have scaling. Androgenetic alopecia will not have
such localised patches, but will be diffuse thinning of the scalp and recession of hairline.

Answer. a

180. A sexually active male comes with lesions on the genitals. On examination, lesions are polypoid
and occasionally bleed. His girlfriend also has similar lesions. On examination, verrucous lesions are
seen on the penile shaft. Histopathology shows koilocytic change. The likely organism is

a. HPV 2 and 3

b. HPV 6 and 11

c. HPV 16 and 18

d. HPV 33 and 35

Solution. b
The disease being described here is condyloma acuminata. Usually, this is caused by the low risk
HPV- namely HPV 6 and 11. Koilocytic change is for HPV infection.

Answer. b
181. All are louse - borne infections except

a. Epidemic typhus

b. Trench fever

c. Lyme disease

d. Pediculosis

Solution. c
Lyme’s disease is caused by bite of a hard tick which transmits borrelia into the skin. The
consequent lesion is called erythema chronicum migrans. Rest all are louse borne infections.
Epidemic typhus is caused by rickettsia prowazekii. Trench fever is caused by bartonella.

Answer. c

182. All the following diseases come on the flexors except

a. Lichen planus

b. Atopic dermatitis in children

c. Intertrigo

d. Inverse psoriasis

Solution. b
Atopic dermatitis in children comes on the extensor surfaces and not on the flexors. Rest all are
flexural situations.

Answer. b
183. A 25-year-old female comes to you with lesions on the lower face as shown. She also reports
blackness of the axillary skin and the neck. She has taken multiple treatments in the past without
response. On examination, there are multiple comedones and papules seen on the lower face. Her neck
and axillary skin show velvety pigmentation. Some terminal hairs are also noticed on her chin. The
treatment for her would best be:

a. Oral isotretinoin

b. Oral Minocycline

c. Cyproterone acetate

d. Topical adapalene

Solution. c
The case is of hormonal acne. Suggestive of this disease is failure to respond to earlier treatment
and the blackness of the axillary skin and the neck which points to acanthosis nigricans.
Hirsutism is also seen on her chin. Cyproterone acetate in combination with ethinyl estradiol
(OCP will be effective in this condition.

Answer. c
184. Which of the following is true about pseudohallucinations?

a. They possess the vivid quality of normal perceptions.

b. They are dependent on external stimuli.

c. They cannot be overcome voluntarily.

d. They arise in inner subjective space.

Solution. d
The term “pseudohallucination” is used when a hallucination is recognized as unreal. According to
Jaspers, pseudohallucinations are not as tangible and real as hallucinations, though they do
appear spontaneously and are discernible from real perception. They can be overcome voluntarily.
They arise in inner space and are not dependent on external stimuli.
Pseudohallucinations are a type of mental image which although clear and vivid lack the
substantiality of perceptions: they are seen in full consciousness and are located in subjective
space.

Answer. d

185. 50-year male presented in emergency in confused state, on examination he is disoriented and
responds poorly to stimuli presented and sometimes the response to the stimuli was nil. After recovery
he had no recollection of the episode. Above clinical scenario is suggestive of which of the following
organic syndrome?

a. delirium

b. stupor

c. subacute delirium

d. twilight state

Solution. B
Disorientation, incoherence of psychic life and some degree of anterograde amnesia are features
of all of these acute organic states. In delirium there is a dream-like change in consciousness, so
that the patient may also be unable to distinguish between mental images and perceptions,
leading to hallucinations and illusions. Usually there is severe anxiety and agitation.
When stupor or torpor is established, the patient responds poorly or not at all to stimuli and after
recovery has no recollection of events during the episode.
In subacute delirium, there is a general lowering of awareness and marked incoherence of
psychic activity, so that the patient is bewildered and perplexed. Isolated hallucinations, illusions
and delusions may occur and the level of awareness varies but is lower at night-time. The
subacute delirious state can be regarded as a transitional state between delirium and organic
stupor.
In twilight states, consciousness is restricted such that the mind is dominated by a small group of
ideas, attitudes and images. These patients may appear to be perplexed but often their behaviour
is well ordered and they can carry out complex actions.

Answer. b
186. A 60-year-old man is brought to the emergency department by his wife for “confusion.” She
reveals that he is a “heavy drinker” and that he has drunk country made liquor almost every day for the
past 30 years. Although he has not changed his alcohol intake significantly, over the past year he has
eaten less, preferring alcohol to large meals. She has noticed a gradual weight loss as a result. His last
drink was earlier that day. Which of the following would be the most likely finding on the mental status
examination of this patient?

a. Confabulation

b. Delusions

c. Elevated affect

d. Fluctuating consciousness

Solution. a
This patient has a long history of heavy, regular alcohol use and likely malnutrition. A common
sequela of this is chronic thiamine deficiency, resulting in Korsakoff syndrome. Korsakoff
syndrome is characterized by an anterograde amnesia; this memory impairment is often (poorly
compensated for by the patient’s confabulation, or filling in the missing memories with false
information.
Wernicke encephalopathy, also called alcoholic encephalopathy, is an acute neurologic
disorder characterized by ataxia (affecting the gait primarily, vestibular dysfunction, confusion,
and a variety of ocular motility abnormalities, including horizontal nystagmus, lateral orbital
palsy, and gaze palsy. These eye signs are usually bilateral but not necessarily symmetrical. Other
eye signs may include a sluggish reaction to light and anisocoria (unequal pupillary size.
Wernicke encephalopathy may clear spontaneously in a few days or weeks or may progress to
Korsakoff syndrome.

Korsakoff syndrome is the chronic amnestic syndrome that can follow Wernicke
encephalopathy. The cardinal features of Korsakoff syndrome are impaired mental syndrome
(especially recent memory and anterograde amnesia in an alert and responsive patient. The
patient may or may not have the symptom of confabulation.

Answer. a
187. A 25-year-old man arrives to the emergency department with gastrointestinal distress, muscle
aches, rhinorrhea, lacrimation, and an anxious mood. He had become addicted on prescription opioids
and now buys pills on the street. He says he works as an accountant and just “wants to kick this thing
once and for all.” Which of the following medications would be most helpful in ameliorating his
symptoms?

a. Disulfiram

b. Clonidine

c. Haloperidol

d. Lorazepam

Solution. b
A rule of thumb regarding opioid withdrawal symptoms is that the shorter the duration of action
of the drug ingested, the more acute and intense the withdrawal symptoms. The longer the
duration of action of the drug being used, the more prolonged, but mild, the symptoms are. An
exception to this rule occurs when an opioid antagonist (eg, naltrexone is given to a person who is
dependent on a long-acting opioid. In this case, the withdrawal symptoms can be severe.
Clonidine can be used to decrease the autonomic and other symptoms of opioid withdrawal, such
as hypertension, tachycardia, sweating, nausea, cramps, diarrhea, lacrimation, and rhinorrhea. It
does not, however, remove the craving for the drug. Blood pressure levels must be monitored
carefully if clonidine is used. Its mechanism of action involves binding to alpha-2 adrenergic
receptors in the locus ceruleus that share potassium channels with opioids, thereby blunting
symptoms of withdrawal.
Methadone can be used instead of, or in addition to, clonidine. It is given orally, in one daily dose,
and is very effective in ameliorating opioid withdrawal syndromes. Unless the patient will
subsequently be placed on methadone maintenance, the dose will need to be slowly tapered over
several days. Buprenorphine (a mixed opioid agonist-antagonist can also be used in lieu of
methadone. It is administered once daily but in a sublingual form and should be gradually tapered
as well.

Answer. b
188. A 75-year-old man is brought in by his daughter for a psychiatric evaluation. He has become
increasingly forgetful over the past year, missing engagements with his children and grandchildren. He
has gotten lost several times driving in his own neighborhood. He has no psychiatric history, but he has
felt lonely since the passing of his wife 14 months ago. His medical history is significant for poorly
controlled hypertension. Which of the following additional features is necessary in order to accurately
diagnose a major Neuro Cognitive Disorder?

a. Agitation

b. Fluctuation in consciousness

c. Loss of independence in activities of daily living

d. Hallucinations

Solution. c
For a diagnosis of major NCD, loss of independence in one or more activities of daily living must
be present. Head CT scan or brain MRI may show lacunar infarcts or microvascular changes in
major vascular NCD, whereas generalized cortical atrophy and ventricular enlargement are the
changes seen in Alzheimer disease . Individuals with major NCDs usually remain alert, whereas
those with delirium display a fluctuation in consciousness . While psychotic symptoms such as
delusions and hallucinations can be seen, they are neither specific to nor necessary for the
diagnosis of a major NCD . Although agitation may be seen with NCDs, it is not a diagnostic
feature.

Answer. c
189. Which of the following is a good prognostic indicator for schizophrenia?

a. Prominent affective symptoms

b. Insidious onset

c. Ventriculomegaly

d. Negative symptoms

Solution.

Answer. a
190. regarding action of local anaesthetic (LA on Na+ channels, which of the following is true?

a. the affinity of LA for the inactivated state of the Na+ channel is higher than its resting state

b. at low frequencies of impulse firing, a phase or use dependant block is seen

c. the cationic form of LA acts on the Na+ channel from outside the axon

d. the unionized form acts from within the axon to inactivate the Na+ channel.

Solution. A
• The Na+ channel exists in three states: resting (R. open (O and inactivated (I.
•The Na+ channel is in the resting state initially , but assumes an open state on stimulation ,
resulting in Na+ influx intracellularly. This triggers the action potential. Subsequently , the
channel is closed by assuming an inactivated state.
• Local anesthetics have a greater affinity for the channel in the open or inactivated state than in
the resting state.
• Local anesthetic binding to open or inactivated channels, or both, is facilitated by
depolarization.
• The fraction of Na channels that have bound a local anesthetic increases with frequent
depolarization (eg, during trains of impulses. This phenomenon is termed use-dependent block .
• Put another way, local anesthetic inhibition is both voltage and frequency dependent, and is
greater when nerve fibers are firing rapidly than with infrequent depolarizations.
• Rate dependant block of cardiac Na+ channels in vivo is the reason that hydrophobic Las are
more toxic than hydrophillic LAs
• It is considered that the uninoized base form of LA crosses the axon membrane and ionizes
intracellularly. This cationic form then acts from within the cell to inactivate the Na+ channel.

Answer. a

191. A patient with history of coronary artery disease presents with pulse rate of 48/min and low . BP
patient has decreased myocardial contractility on Echo . Which of these anesthetic agents should be
used for induction of anesthesia?

a. Ketamine

b. Dexmedetomidine

c. Etomidate

d. Fentanyl

Solution. C
• Etmidate is a imidazole derivative iv induction agent.
• It is milky white in colour
• It is diluted in propylene glycol and is painful on injection.
Effects on the Cardiovascular System
• The hemodynamic stability seen with etomidate is caused by its lack of effect on the sympathetic
nervous system and on the function of the baroreceptor.
• Etomidate has proven useful in patients with valvular or ischemic heart disease who are
undergoing noncardiac surgical procedures and in patients with poor cardiac function.
• The myocardial oxygen supply-to-demand ratio is well maintained.

Answer. c
192. The circled part of the picture reflects;

a. dead space air

b. mixed air

c. inhalation

d. alveolar air

Solution. D
• Capnography is measurement of end tidal carbondioxide concentration
• Its normal value is 35-45 mmhg
• Infrared spectroscopy is most common method of estimating ETCO2
• This a typical capnographich waveform

Phase I: dead space gases ( No CO2


Phase II: expiratory upstroke ( mixed dead space gas + alveolar CO2
Phase III: plateau phase ( alveolar gas
Phase 0: inspiration

Answer. d
193. A man being treated for severe asthma experiences an episode of life-threatening tachycardia
requiring emergency treatment. Which drug is most likely responsible for this adverse effect?

a. ipratropium

b. formoterol

c. cromolyn

d. montelukast

Solution. B
Formoterol is a long-acting β2-agonist that may cause tachycardia and death in asthmatic
patients.
Although ipratropium may occasionally cause tachycardia, the drug is poorly absorbed after
inhalation and is less likely to cause severe tachycardia than are the β2-agonists.

Answer. b

194. Sugammadex, used to reverse an overdose with neuromuscular blockers, has a unique mechanism
of action best described as which one of the following?

a. agonist at cholinergic acetylcholine receptors

b. antagonist at cholinergic acetylcholine receptors

c. forms an inactive drug complex

d. inhibits acetylcholinesterase

Solution. C
Sugammadex is indicated for the rapid reversal of the effects of steroidal neuromuscular blocking
agents, like rocuronium and vecuronium It is a modified γ-cyclodextrin compound that binds with
rocuronium or
vecuronium to form an inactive drug complex that is rapidly excreted in the urine.

Answer. c
195. Which one of the following drug useful in glaucoma, acts on letter mentioned B in this diagram?

a. Pilocarpine

b. Brimonidine

c. Acetazolamide

d. Bimatoprost

Solution. D

Answer. d
196. After being stung by a bee, a child is given an intramuscular injection of epinephrine. Which
action would lead to bronchodilation?

a. increased cAMP levels

b. increased cyclic guanosine monophosphate (cGMP levels

c. increased IP3 levels

d. calcium influx

Solution. A
The preferred treatment for severe hypersensitivity reactions (anaphylaxis is epinephrine. The
drug causes bronchodilation by activation of β2-adrenoceptors in bronchial smooth muscle,
leading to increased cAMP levels and smooth muscle relaxation. cGMP

Answer. a

197. Match the following regarding antihypertensives and its adverse effects
DRUG ADR

a. Nifedipine p. hypothyroidism

b. Hydralazine q. haemolytic anemia

c. Sodium nitroprusside r. gum hypertrophy

d. Methyldopa s. SLE

Solution. C
Nifedipine- GUMHYPERTROHY
Hydralazine causes SLE
Sodium nitroprusside- hypothyroidism
Methyldopa - autoimmune haemolytic
anemia

Answer. a-r,b-s,c-p,d-q

198. Amiloride is useful in cystic fibrosis

a. True

b. False

Answer. a
199. Chlorothiazide is useful in liddle’s syndrome

a. True

b. False

Answer. b

200. Torsemide is the longest acting loop diuretic

a. True

b. False

Answer. a

201. Large intravenous doses of furosemide may cause tinnitus, vertigo, and hearing loss

a. True

b. False

Answer. a

202. Acetazolamide used to increase the renal excretion of weakly acidic drugs such as amphetamine

a. True

b. False

Solution. c: furosemide. Loop-acting diuretics such as furosemide can cause ototoxicity more
frequently than other diuretics. Of the loop-acting diuretics, edthacrynic acid is particularly prone
to cause adverse effects such as tinnitus (ringing in the ears, vertigo, hearing impairment, and ear
pain
acetazolamide. Carbonic anhydrase inhibitors such as acetazolamide increase renal sodium
bicarbonate excretion and alkalinize the urine. Urine alkalinization increases the ionization of
weakly acidic drugs such as amphetamine and thereby increases their renal excretion.

Answer. a

203. Comparing to Filgrastim, pegfilgrastim is short acting


a. True

b. False

Answer. b

204. Oprelvekin is useful for treating thrombocytopenia due to cancer chemotherapy

a. True

b. False

Answer. a

205. Idarucizumab is useful for reversal of action of rivoraxaban

a. True

b. False

Answer. b

206. Eptifibatide is a inhibitor of active Stuart factor

a. True

b. False

Answer. b

207. TENECTEPLASE is the Fibrin specific thrombolytic agent

a. True

b. False

Solution. B: Comparing to Filgrastim, pegfilgrastim is longer acting agent Oprelvekin is a IL- 11


analogue, useful for treating thrombocytopenia due to cancer chemotherapy- Idarucizumab is
useful for reversal of action of dabigatran Eptifibatide is a inhibitor of GPIIB/IIIA inhibitor
TENECTEPLASE is the Fibrin specific thrombolytic agent

Answer. a
208. Match the following drug and its indications

DRUG INDICATION

a. Haloperidol p. Parkinson disease psychosis

b. Aripiprazole q. Tardive dyskinesia

c. Pimavanserin r. Gilles de la Tourette syndrome

d. Valbenazine s. irritability in autistic children.

Solution. C
Haloperidol Gilles de la
Tourette syndrome
Aripiprazole Birritability in
autistic children.
Pimavanserin Parkinson disease
psychosis
Valbenazine DTardive
dyskinesia

Answer. a-r,b-s,c-p,d-q

209. Danazol is used to treat hereditary angioedema due to deficiency of first-component esterase
inhibitor

a. True

b. False

Answer. a

210. letrozole has been used to treat infertility in anovulatory women

a. True

b. False

Answer. a

211. Bazedoxifene isapproved for the treatment of moderate to severe VMS

a. True

b. False

Answer. a
212. Ospemifene is indicated for the treatment of moderate to severe dyspareunia

a. True

b. False

Answer. a

213. Oxandrolone is approved as an adjunct drug to increase weight gain after weight loss of surgery

a. True

b. False

Solution. D : Danazol is used to treat hereditary angioedema due to deficiency of first-component


esterase inhibitor letrozole has been used to treat infertility in anovulatory women Bazedoxifene
isapproved for the treatment of moderate to severe VMS Ospemifene is indicated for the
treatment of moderate to severe dyspareunia Oxandrolone is approved as an adjunct drug to
increase weight gain after weight loss of surgery

Answer. a

214. Which antibiotic is highly ionized in body fluids and must be given parenterally for systemic
infections?

a. doxycycline

b. azithromycin

c. clindamycin

d. tobramycin

Solution. D: tobramycin. Aminoglycoside antibiotics such as tobramycin are ionized in body


fluids and must be given parenterally for systemic infections

Answer. d
215. Which drug exerts a fungicidal effect by inhibiting synthesis of the fungal cell wall?

a. micafungin

b. posaconazole

c. nystatin

d. terbinafine

Solution. A
micafungin. Micafungin, caspofungin, and anidulafungin are echinocandin drugs that inhibit the
synthesis of β-(1,3-D-glucan, a component of the fungal cell wall.
Posaconazole and other azole drugs inhibit a P450 enzyme that catalyzes the formation of the
lanosterol component of the fungal membrane.
nystatin and amphotericin B bind to ergosterol and create pores in fungal membranes.
Terbinafine inhibits squalene epoxidase and the formation of the precursor to lanosterol,
squalene-2, 3-oxide

Answer. a

216. match the following regarding drug and its target

DRUG TARGET

a. Alemtuzumab p. epidermal growth factor

b. Ipilimumab q. programmed death-1 (PD-1

c. Cetuximab r. cytotoxic T-lymphocyte–associated antigen 4

d. Nivolumab s. binds to CD52

Solution. B
Alemtuzumab binds to CD52
Ipilimumab cytotoxic T-
lymphocyte–associated antigen 4
Cetuximab epidermal
growth factor
Nivolumab programmed
death-1 (PD-1

Answer. a-s,b-r,c-p,d-q

Test Answer

1 (d) 2 (c) 3 (a) 4 (b) 5 (c) 6 (b) 7 (d) 8 (c)


9 (b) 10 (a) 11 (c) 12 (b) 13 (a) 14 (c) 15 (a) 16 (b)
17 (d) 18 (b) 19 (c) 20 (a) 21 (c) 22 (d) 23 (a) 24 (b)
25 (c) 26 (d) 27 (a) 28 (d) 29 (a) 30 (b) 31 (c) 32 (d)
33 (a) 34 (c) 35 (b) 36 (a) 37 (c) 38 (d) 39 (c) 40 (d)
45 (a-p,b-r,c-- 46 (a-s,b-p,c--
41 (c) 42 (c) 43 (b) 44 (a) 47 (c) 48 (d)
,d-s) ,d-t)
56 (a-r,b-s,c--
49 (a) 50 (b) 51 (d) 52 (a) 53 (d) 54 (b) 55 (b)
,d-p)
61 (a-s,b-r,c--
57 (b) 58 (c) 59 (c) 60 (d) 62 (b) 63 (b) 64 (d)
,d-p)
65 (a-q,b-s,c--
66 (d) 67 (c) 68 (b) 69 (d) 70 (b) 71 (c) 72 (d)
,d-r)
73 (a) 74 (a) 75 (c) 76 (d) 77 (b) 78 (c) 79 (d) 80 (d)
81 (c) 82 (a) 83 (d) 84 (b) 85 (c) 86 (b) 87 (c) 88 (a)
89 (d) 90 (d) 91 (a) 92 (c) 93 (a) 94 (d) 95 (d) 96 (b)
104 (a-s,b-r-
97 (c) 98 (a) 99 (a) 100 (b) 101 (d) 102 (a) 103 (b)
c-q,d-p)
105 (d) 106 (a) 107 (b) 108 (c) 109 (d) 110 (c) 111 (b) 112 (b)
113 (c) 114 (c) 115 (a) 116 (c) 117 (d) 118 (c) 119 (a) 120 (a)
121 (a) 122 (d) 123 (b) 124 (d) 125 (d) 126 (b) 127 (a) 128 (b)
129 (a) 130 (a) 131 (b) 132 (a) 133 (a) 134 (a) 135 (d) 136 (b)
137 (a) 138 (a) 139 (b) 140 (a) 141 (a) 142 (d) 143 (a) 144 (c)
149 (a-q,b-p-
145 (b) 146 (c) 147 (a) 148 (b) 150 (b) 151 (a) 152 (b)
c-s,d-r)
153 (a-q,b-p- 159 (a-r,b-p-
154 (a) 155 (c) 156 (b) 157 (b) 158 (a) 160 (d)
c-s,d-r) c-s,d-q)
161 (b) 162 (b) 163 (c) 164 (c) 165 (b) 166 (a) 167 (b) 168 (a)
169 (d) 170 (c) 171 (c) 172 (b) 173 (a) 174 (d) 175 (b) 176 (d)
177 (b) 178 (d) 179 (a) 180 (b) 181 (c) 182 (b) 183 (c) 184 (d)
185 (b) 186 (a) 187 (b) 188 (c) 189 (a) 190 (a) 191 (c) 192 (d)
197 (a-r,b-s-
193 (b) 194 (c) 195 (d) 196 (a) 198 (a) 199 (b) 200 (a)
c-p,d-q)
208 (a-r,b-s-
201 (a) 202 (a) 203 (b) 204 (a) 205 (b) 206 (b) 207 (a)
c-p,d-q)
216 (a-s,b-r-
209 (a) 210 (a) 211 (a) 212 (a) 213 (a) 214 (d) 215 (a)
c-p,d-q)

You might also like